Вы находитесь на странице: 1из 61

TEST OF ENGLISH LANGUAGE

COMMON ERRORS

1. Some people are believing that English is difficult to learn.


Ans (2): The stative verbs (agree, believe, consider, doubt, differ etc.,) are used to denote permanent mental state.
It is why these verbs are generally not used in Progressive / Imperfect Tense. Hence, ‘are believing’ should be
replaced by ‘believe’.

2. We should not avoid reading English at regularly intervals. (4)/No error (5)
Ans (4): The word interval is a Noun. To qualify a Noun, we use an Adjective, not an Adverb. Hence, ‘at regularly
intervals’ should be replaced by ‘at regular intervals’. The word ‘regularly’ is an Adverb whereas ‘regular’ is an
Adjective

5. Accidents usually (1)/result into (2)/ negligence, ignorance (3)/recklessness or carelessness. (4)/No error (5)
Ans (2): The sentence shows cause. Hence, ‘results into’ should be replaced by ‘result from’.

Look at the sentences:


The failure of the company results from bad management. The cyclone has resulted into many thousands of deaths.
6. He asked me that (1)/what my total investment was (2)/during the (3)/last five years. (4)/No error (5)
Ans (4): When Interrogative sentences are written in Indirect Speech, conjunction ‘that’ is not used with Wh-
questions.

Look at the sentences:


He said to me, “Where do you live?” (Direct) He asked me where I lived. (Indirect) Hence ‘he asked me what my
…’ is the correct sentence.

7. This institution had founded (1)/by some noble souls (2)/for the welfare of the poor (3)/and disabled people.
(4)/No error (5)
Ans (1): Here Passive Voice (Simple Past) should be used.
Hence, ‘the institution was founded…..’ is the correct sentence.

8. Every driver should take (1)/abundant precautions (2)/while driving on (3)/ an unfamiliar road. (4)/No error (5)
Ans (5): No error
9. His good qualities include (1)/his ability to take (2)/quickly and (3)/appropriate decisions. (4)/ No error (5)
Ans (3): The word quickly (Adverb) should be replaced by the word quick (Adjective).
10. in order to (1)/ ensure quick service (2)/ please give us all the required (3)/ details as quick as possible. (4)/ No
error (5)
Ans (4): As soon as is the correct form of correlative. Hence ‘as quick as’ should be replaced by ‘as soon as’.
11. The club has lost (1)/ a lot of business (2)/ because of poor (3)/ maintained facilities. (4)/ No error (5)
Ans (3): Replace ‘because of poor’ by ‘because of poorly’, because the word poorly is an Adverb which modifies the
word ‘maintained’ (Adjective).
12. The village has electricity (1)/ today only because (2)/ of the efforts of a (3)/ few dedicated engineers. (4)/ No
error (5)
Ans (5): No error
13. The company has appointed (1)/ consultants to help them (2)/ increase its revenue (3)/ and improve its
financial position. (4)/ No error (5)

Ans (2): Here, in the group of words ‘consultants to help them’, the word ‘them’ has been wrongly used for the
company. It is correct to use Objective Case ‘it’ (Pronoun) instead.
14. The cost of the new (1)/ machines is likely to (2)/ be so high as ten. (3)/times the existing ones (4)/ No error (5)

Ans (3): As ……. as….is used when comparing two people or things. Therefore, ‘be so high as ten’ should be
replaced by ‘be as high as ten’.
15. The NGO provides (1)/ free daily meal to them (2)/ who live below the (3)/ poverty line in rural areas. (4)/ No
error (5)

Ans (5): No error.

16. He reports (1)/ suggest that (2)/ the project is (3)/ behind schedule. (4)/ No error (5)

Ans (1): The possessive pronoun of ‘he’ is ‘his’.


Hence, replace ‘He reports’ by ‘His reports’.
17. He is great leader (1)/ who inspired people (2)/ to achieve freedom (3)/ through non- violent means. (4)/ No
error (5)

Ans (1): The sentence express past time. Hence use ‘He was great leader………..’.
18. An estimate forty (1)/ lakh workers will (2)/ benefit from the (3)/ amendment to the Act. (4)/ No error (5)

Ans (1): An estimated forty….. will be a correct sentence. It is better to use Adjective form of the word ‘estimate’.
19. What satisfies a teacher (1)/ the most is the (2)/ excitement of students when (3)/ they learnt something new.
(4)/ No error (5)

Ans (4): When we express a truth or habit, it is correct to use Simple Present.
Hence, ‘they learn something new’ will be correct usage.
20. She is the first woman (1)/ to be appointed as president (2)/ since the company (3)/ was established from 1950.
(4)/ No error (5)

Ans (4): Replace ‘was established from 1950’ by ‘was established in 1950’. Preposition ‘from’ is generally used to
express the start of time.
Look at the sentence:
He is available at home from 7 a.m to 8 a.m.
21. Some of our staff is worried (1)/that as soon as (2)/the project is over they (3)/ will lose their jobs. (4)/ No error
(5)

Ans (1): As the subject of the sentence is plural, it will take Plural Verb. Hence, ‘Some of our staff are’……….. Will
be correct usage.
22. If you decide to hold (1)/ the function in Kolkata (2)/ not much of us (3) / will be able to attend. (4)/No error
(5).

Ans (3): Replace ‘not much of us’. ‘Much’ is used before Uncountable Noun and many is used before Countable
Noun.
23. In case you need five people (1)/ to run a branch you (2)/ should be selected (3)/ ten since some may leave. (4)/
No error (5)

Ans (3): It will be proper to use Active Voice. Hence, replace ‘should be selected’ by ‘should select’.
24. Although he approached (1)/many private businesses to invest (2)/ in his printing business (3)/ nobody was
interested (4)/No error (5).

Ans (5): No error.


25. How can you give up (1)/ this job when you are (2)/ just about to be (3) / appointed for General Manager? (4)/
No error (5).

Ans (3): Replace group of words ‘just about to be’ by ‘just to be’ or ‘about to be’.
26. Base on the research (1) / we have conducted in (2)/ different parts of the country (3)/ this scheme will be
successful (4). No error (5).

Ans (1): Replace group of words ‘Base on the research’ by ‘Based on the research’. It is proper to use Past Participle
form of Verb ‘base’
27. I want to share (1)/ my experience with you (2)/ though you will (3)/ benefit from it (4)/. No error (5)

Ans (3): Replace connective


28. He used to advise (1)/ his students to do (2)/ their work serious if they (3)/ wanted to achieve their goals (4).
No error (5).

Ans (3): Replace ‘their work serious if they’ by ‘their work seriously if they’. An Adverb should be used to modify a
Verb.
29. Any of these branches do (1)/ not require more than (2)/ two employees since they (3)/ have been recently
established (4). No error (5)

Ans (1): ‘Any of these branches’ expresses singular subject. Hence, it will take Singular Verb.
30. He encouraged us to (1)/ think but none of the (2)/ suggestions we made was (3)/ discussed at the meeting (4).
No error (5).

Ans (5): No Error.


31. The main advantage of (1)/ investing such schemes (2)/ is that you will not (3)/ have to pay any taxes (4). No
error (5)

Ans (2): The word ‘invest’ takes preposition ‘in’. Hence, ‘investing in such schemes’ will be correct usage.
32. That customer has (1)/ written a letter (2)/ thanks the manager (3)/for her timely help. (4). No error (5)

Ans (3): Here, Gerund should be used. Hence, replace ‘thanks the manager’ by ‘thanking the manager’.
33. Our company was recently (1)/ award the contract (2)/ to construct two major (3)/ bridges in the state. (4). No
error (5)

Ans (2): The structure of sentence in Simple Past in Passive Voice is: Subject + was/were + V3. Hence, replace
award the contract by ; ‘awarded the contract’.
34. Accordingly the circular (1)/ that was issued yesterday RBI (2)/ has raised the rate at (3)/ which it lends to
banks. (4). No error (5)

Ans (1): ‘According to / In accordance with’ is a correct idiomatic expression. Hence, replace ‘Accordingly the
circular’ by ‘According to the circular’.
35. Their company is going to incur (1)/ heavy losses this year because (2)/ of the big discounts they (3)/ offers to
customers. (4). No error (5)

Ans (4): Here, ‘they’ is in Plural Number that will agree with a Plural Verb. Hence, ‘offer to customers’ should be
used.
36. A large number of policy (1)/ have lapsed because many (2)/ people have not paid (3)/ their installments on
time. (4). No error (5)

Ans (1): ‘A large number of ‘is followed by Plural Noun. Hence, ‘A large number of policies’ will be a correct usage.
37. The Foreign Investment Promotion Board is (1)/ the government body who (2)/ regulates investment received
(3)/ by Indian factories from foreign countries. (4). No error (5)

Ans (2): For non living Nouns ‘which’ is used as reflexive Pronoun. Hence, ‘the government body which’ will be
correct usage.
38. The final decision can (1)/ be taken only after (2)/ we consider all (3)/ the panel suggestions. (4). No error (5)

Ans (5): No error.


39. If his aim is to (1)/ buy a house in the (2)/ next three years he (3)/ should start immediate saving. (4). No error
(5)

Ans (4): The word immediately is an Adjective Verb which modifies a Verb. Hence, ‘should start saving
immediately’ will be correct usage.
40. Can you explain why (1)/ you were late this morning (2)/ when we had to make (3)/ such an important
presentation? (4). No error (5)

Ans (2): Look at the following structure of Interrogative sentences in Past Continuous: What/why/where
+was/were + V4/v-ing) Adjective
Hence, ‘were you late this morning’ will be correct usage.
41. In order to (1)/attract tourists many (2)/ hotels have been offered (3)/ attractive deals and discounts. (4)/ No
error (5)

Ans (3): The structure of the sentence in Present Perfect Continuous is: Subject + has/ have been +V – ing
Hence, “hotels have been offering” will be correct usage. Here ‘Simple Present’ can also be used.
42. The Board is likely (1)/ to take its time (2)/ to examine the facts (3)/ before giving their decision. (4)/ No error
(5)

Ans (4): The possessive case of the subject ‘The Board’ will be ‘its’. Hence, before giving its decision’ will be a
correct usage.
43. How can we open (1)/ these branches on time (2)/ if we have not (3)/ yet obtained the licenses? (4)/ No error
(5)

Ans (5): No Error


44. One of the disadvantage (1)/ of fixed deposit schemes (2)/ is that banks offer (3)/ low rates of interest. (4)/ No
error (5)

Ans (1): ‘One of’ is followed by Plural Noun/Pronoun.


Hence, ‘One of the disadvantages’ will be a correct usage.
45. According to experts, it is (1)/ necessarily for you to save (2)/ and invest at least twenty-five (3)/ percent of
your monthly income. (4)/ No error (5)

Ans (2): The word necessarily is an Adverb while Adjective ‘necessary’ should be used.
46. The manager has not (1)/ yet replied to the (2)/ customer letter asking (3)/ about her loan application. (4)/ No
error (5)

Ans (3): Replace ‘Customer letter asking’ by ‘Customer’s letter asking’.


47. While the formation of RBI (1)/ was approved in March, 1934 (2)/ it was inaugurated (3)/ only in April 1, 1935.
(4)/ No error (5)

Ans (4): Here, a particular date has been mentioned. Hence, replace ‘only in April 1,1935’ by ‘only on April 1,1935’.
48. The Insurance Regulatory Development Authority has (1)/ asked all insurance (2)/ companies submitting their
(3)/ balance sheets by June. (4)/ No error (5)

Ans (3): Here infinite –to + V1 –should be used. Hence, replace ‘companies submitting their’ by ‘companies to
submit their’.
49. Arun’s father has been ill (1)/ since last year and Arun (2)/ has spend his entire (3)/ savings on his treatment.
(4)/ No error (5)

Ans (3): The structure of sentence in Present Perfect is: Subject + has / have + V3
Hence, replace ‘has spend his entire’ by ‘has spent his entire’.
50. There are a large number (1)/ of Chinese workers which (2)/ are employed in (3)/ software companies in India.
(4)/ No error (5)

Ans (2): For living Noun/Pronoun Reflexive Pronoun ‘who’ should be used. Hence, ‘of Chinese workers who’
should be used.
51. My friend lives (1)/ in a beautiful house (2)/ not more than (3)/ five minutes from the beach. (4). No error (5).

Ans (4): Here, five kilometers from beach should be used. Here, indicative of distance should be used
52. If you breaks the law, (1)/ you must be prepared (2)/ to suffer (3)/ the consequences. (4). No error (5).

Ans (1): Second person agrees with a Plural Verb. Hence, ‘ if you break the law ……’ should be used here.
53. I requested my friend (1)/ to come (2)/ and shopping (3)/ with me. (4). No error (5).

Ans (3): Here, infinitive i.e. and shop … should be used because to come has been used as infinitive.
54. The passage is (1)/ more difficult (2)/ that I am unable (3)/ to comprehend it. (4). No error (5).

Ans (2): Here, so difficult should be used.


Look at the sentences: He is too weak to walk. He is so weak that he is unable to walk.
55. Janet is determined (1)/ to be success (2)/ in whatever field (3)/ she chooses. (4). No error (5).

Ans (2): Here, Past Simple should be used.


56. Handsome is usually used (1)/ of men but beautiful is not (2)/ usually used to talk (3)/ about man’s
appearance. (4)/ No error (5)

Ans (4): ‘About a man appearance’ will be replaced by ‘about a man’s appearance’.
57. Payal kept her drum (1)/ besides her (2)/ always and (3)/ she played wisely. (4)/ No error (5)

Ans (2): ‘Besides her’ is replaced by ‘beside her’.


58. Either you (1)/ or your friend (2)/ has stolen (3)/ my wrist watch. (4)/ No error (5)

Ans (5): When two Subjects are joined by Either ….or / Neither … Nor then Verb should be according to the second
Subject.
For example, Either you or your friend is guilty.(✓) Either you or your friend are guilty. (X)
59. My son has (1)/ bought four pens (2)/ who are (3)/ red in color. (4)/ No error (5)

Ans (3): Use ‘that are’ in place of ‘who are’. There remains a close relation between Relative Pronoun and its
Antecedent in a sentence.
The Noun and Pronoun that are used before who/which/that are called Antecedents. When Antecedent refers to
human being, then who or that are used. On the other hand, if Antecedent refers to non-livings, small animals or
plants. Then we use which or that.
The student, who are playing there with my children, is Sarika’s son. (√) The student, which are playing there with
my children, is Sarika’s son. (X) The watch who is on the table is mine. (X)
The watch that / which is on the table is mine. (√)
60. If you look up ‘tea’ in a cookery book (1)/ you may find a little instructions (2)/ that give you no help (3)/ on
several important points. (4)/ No error (5)

Ans (2): Use ‘a few instructions’ in the place of ‘a little instruction’. ‘A little’ is used with a Uncountable Nouns
while ‘a few’ is used with Countable Noun.
61. The Chinese tea is (1)/ not very costly (2)/ and we can drink (3)/ it without milk. (4)/ No error (5)

Ans (5): No error.


62. Alaripu is an invocatory item (1)/ where the dancer (2)/ offers salute (3)/ to the lord. (4)/ No error (5)

Ans (3): Use ‘offers salutation’ in the place of ‘offers salute’. ‘Salute’ is a Verb while ‘salutation’ is a Noun.
63. He served the country (1)/ with heart and soul (2)/ but he got (3)/ only a few rewards. (4)/ No error (5)

Ans (2): Use ‘heart and soul’ in the place of ‘with heart and soul’. ‘Heart and soul’ is an idiom which means ‘wholly
or devotedly’. Therefore, no Preposition will be used before it. The sentence would be:
He served the country heart and soul but he got only a few rewards.
64. With rise in population (1)/ man needed more space (2)/ for growing crops (3)/ and for building houses. (4)/
No error (5)

Ans (5): No error


65. Which (1)/ do you prefer (2)/ more meat (3)/ or fish? (4)/ No error (5)

Ans (3): The word ‘more’ should not be used. ‘Prefer’ is itself a comparative Degree.
66. The principle that (1) / a pen is mightier (2) / then a sword (3) / was known to early Greeks (4) / No error (5)

Ans (3): The group of words ‘then a sword’ be replaced by ‘than a sword’. In comparative degree we use ‘than’, not
‘then’.
67. Martha missed her dog (1) / who died of a (2) / skin infection at a (3) / very young age. (4) / No error (5)

Ans (1): Use ‘Martha is missing….’ Or ‘Martha misses….’.


68. There was such (1) / a crowd that (2) / he would not (3) / find his brother. (4) / No error (5)

Ans (3): The sense of the sentence shows the inability. Hence, it will be proper to use ‘he could not’ instead of ‘he
would not’.
69. Having managed the (1) / team of two (2) / years, he knew (3) / his players well. (4) / No error (5)

Ans (2): It is proper to use preposition ‘for’ as it shows period of time.


70. Because of heavy (1) / rains the trains (2) / are disable to (3) / run on time. (4) / No error (5)

Ans (3): Replace ‘are disabled to’ by ’are unable to’.


Look at the sentence: I tired to meet him but was unable to.
71. The state government (1) / has send reports (2) / to the centre (3) / on the situation. (4) / No error (5)

Ans (2): In Present Perfect Tense the structure of the sentence is: Subject _ Have/has + V3
Hence, the state government has sent repots …. Will be correct sentence.
72. Whether a person (1) / tales to smoking (2) / depends a lot (3) / in his social network. (4) / No error (5)

Ans (4): Use preposition ‘on’ in place of ‘in’. Look at the sentences: He is the sort of person you can depend on.
It would depend on the circumstances.
73. Every home and office (1) / needs the latest equipment (2) for keep up (3) / with times (4) / No error (5)

Ans (3): The correct infinitive will be ‘to keep up’.


74. The idea of turning (1) / invisible with the (2) / swish of clock have (3) / always fascinated scientists. (4) / No
error (5)

Ans (3): The subject of the sentence is ‘The idea’ which is singular and hence it will take Singular Verb.
Hence, The idea of turning invisible with the swish of clock has … will be a correct sentence.
75. She had seen (1) / the approaching truck (2) / and had jumping (3) / out of the way. (4) / No error (5)

Ans (3): In Past Perfect we use Past Participle form of a Verb. Hence, replace ’and had jumping’ by ‘had jumped’.
76. People who intend (1)/to visit the tourist spots (2)/are always thrilling (3)/to see the scenario here (4). / No
error (5)

Ans (3): Replace groups of words ‘are always thrilling’ by ‘are always thrilled’. In Passive Voice, the Past Participle
form of Verb is used.
77. In such delicate matters, (1)/we often go with (2) /his advice as he has (3)/been handling such cases effectively
(4)/. No error (5).

Ans (5): No error


78. You should think that (1)/ of all the possibilities (2)/before you take (3)/ any decision (4)/.No error (5)

Ans (1): The use of Conjunction ‘that’ is Superfluous.


79. He was too tired that (1)/ he could not cross (2)/ the street even with (3)/the help of a porter (4)/. No error (5).

Ans (1): Replace group of words ‘He was too tired that’ by ‘He was so tired that’
Look at the sentences: He is so tired to walk. He is so tired that he cannot walk.
80. Your desire to (1)/meet the President (2)/ without prior appointment (3)/ are impossible to be satisfied (4)/. No
error (5).

Ans (4): Replace group of words ‘Your desire to’ by ‘You desire to’ or ‘your desire is to ‘or ‘I desire to’.
81. Whenever a man attain fame, (1)/ his personal qualities are (2)/ imitated by other who (3)/ are close to him
(4)./No error (5)

Ans (1): Here, Subject ‘a man’ is singular. Hence whenever a man attains fame …. Will be correct sentence.
82. Rivers, mountains and deep forests (1)/ are the places (2)/ mostly like by (3)/people living in urban areas
(4)/.No error (5).

Ans (3): Replace group of words ‘mostly like by’ by ‘most liked by’. The sentence is in Passive voice.
83. When we visited his office (1)/ we found that (2)/ he was sipping coffee (3)/ with some of his colleagues (4)/.
No error (5).

Ans (4): Active Voice will be used. Hence, similar other methods can help’ should be used.
84. For giving up (1)/ the bad habit of smoking (2)/ use of chewing gum or (3)/ similar other method can be helped
(4)/. No error (5).

Ans (5): No error


85. His obviously reluctance (1) / was viewed seriously by (2)/ his superiors and (3)/ he was suspended (4) /. No
error (5).

Ans (1): Replace groups of words His obviously (Adverb) reluctance by His obvious (Adjective) reluctance.
86. I cannot able to teach you (1)/ how to learn the guitar (2)/ if you do not (3)/ show any interest in music. (4)/ No
Error (5).

Ans (1): Here, I cannot be able to teach you... should be used.


Look at the sentences: I cannot teach you. I cannot be a teacher (Noun). I cannot be rich (Adjective).
87. The researcher could not (1)/ understand the local language (2)/ of the state and so, (3)/ found it difficult to
study in the people. (4)/ No Error (5).

Ans (4): It is superfluous to use ‘in’ here. Hence, found it difficult to study the people/on the people … should be
used.
Look at the sentences:
He studied her face thoughtfully. He was studying the menu.
88. Since it was (1)/ a very cold and windy day (2)/ I asked my friend (3)/ to off the fan. (4)/ No Error (5).

Ans (4): Here, to switch off the fan … should be used, Phrase switch off/on means: to turn a light, machine etc,
off/on by pressing a button or switch.
Look at the sentences:
Please switch the lights off as you leave. How do you switch this things on?
89. The nurses want (1)/ not only a pay increased (2)/ but also reduce (3)/ in working hours as well. (4)/ No Error
(5).

Ans (3): Here, but also reduction should be used.


90. You cannot expect the students (1)/ to blame on the teacher (2)/ for getting low marks (3)/ in the examination.
(4)/ No Error (5).

Ans (2): Here, to blame the teacher … should be used.


Look at the sentences:
He doesn’t blame anyone for the fire. He put the blame on me for being late.
91. I live very (1)/ close by to the train station, (2)/ so travelling to work (3)/ is very convenient for me. (4)/ No
Error (5).

Ans (2): Here, close by/to the railway station … should be used.
Look at the sentences: The route passes close by the town.
The picture looks very different when you see it close to.
92. There is appearing (1)/ to be a shortage (2)/ of skilled staff (3)/ in the organisation. (4)/ No Error (5).

Ans (1): It is not proper to use appear (Verb) in Progressive Tense. Hence, there appears.
…should be used.
93. The patient lodged (1)/ a complaint against (2)/ the doctor for (3)/ its negligence. (4)/ No Error (5).

Ans (2): Here, his/her negligence should be used. ‘Its’ is used for non-living things.
94. My colleague informed (1)/ to her boss that (2)/ she needed more time (3)/ to complete the assignment. (4)/ No
Error (5).

Ans (2): Here, her boss that … should be used.


Look at the sentences: Please inform us of any change of address.
95. At first, (1)/ I would like to (2)/ thank you all (3)/ for inviting me here today. (4)/ No Error (5).

Ans (5): No error.


96. He was awfully dissatisfy (1)/ with the arrangements (2)/ made for him (3)/ by the security guards. (4)/ No
error (5).

Ans (1): Here, it will be proper to write ‘he was awfully dissatisfied’. The word ‘dissatisfied’ is used in this form as
an ‘Adjective’. In the sentence, the Verb ‘dissatisfy’ has been wrongly used.
97. I wanted to be (1)/ out of the way (2)/ when she was (3)/ likely for receiving the letter. (4)/ No error. (5)

Ans (4): Use of Gerund is wrong here. It is correct to use an infinitive. Hence, replace ‘likely for receiving the letter’
by ‘likely to receive the letter.
98. We have rarely come (1)/ across any programme (2)/ which does not show (3)/ violent in some from. (4)/ No
error (5).

Ans (4): Replace ‘violent in some from’ by ‘violence in some from’. The word ‘violent’ is an Adjective where as the
word ‘violence’ is a Noun. The sense of the sentence suggests so.
99. A lot of people (1)/ have involved in (2)/ the national mission (3)/ to eradicate illiteracy. (4)/ No error (5).

Ans (2): It is improper to use Present Perfect her. Hence, a lot of people are involved in … will be a correct sentence.
100. They failed in (1)/ their plan only because (2)/ they could not (3)/ give timely attention to it. (4)/ No error (5).

Ans (5): No error.


101. It is absolutely (1)/ necessarily to participate (2)/ in any project (3)/ for social upliftment. (4) / No error (5).

Ans (2): The word ‘necessarily’ is an Adverb. It will be proper to use an Adjective here. Hence, it is absolutely
necessary …. Will be a correct sentence.
102. We admire your (1)/ contribution to (2)/ this project and wishing (3)/ you a fruitful tenure .(4)/ No error (5).

Ans (3): The first part of the sentence in Simple Present. Hence, it is improper to use ‘wishing’ (Progressive) and
conjunction ‘and’. Hence, we admire your contribution to this project and wish
… will be correct sentence.
103. He always makes (1)/ it a point to (2)/ listen to the what (3)/ others say. (4)/ No error (5).

Ans (3): Use of article ‘the’ is incorrect.


104. Many of us (1)/ have been undergone (2)/ the agony of losing (3)/ someone important to us. (4)/ No error (5)

Ans (2): Use of Passive voice is incorrect. Hence, many of us have undergone …. Will be a correct sentence.
105. Put the work aside (1)/ she got up from her seat (2)/ and marched off without (3)/ looking at the visitor. (4)/
No error (5).

Ans (1): It should be used Present Participle/ Past Participle here. Hence, ‘putting the work aside/having put the
work aside’ will be correct usage.
106. He made a deliberately (1)/ attempt to malign (2)/ his friend and to (3)/ benefit from it. (4)/ No error (5).

Ans (1): The word ‘deliberately’ is an Adverb. As an Adjective qualifies a Noun, it will be correct to use ‘deliberate’
(Adjective) here.
107. Once he was satisfied by himself, (1)/ he started thinking (2)/ about what had gone wrong (3)/ and why it had
done so. (4)/ No error (5).

Ans (4): Use ‘and why he had done so’.


108. When there is a lot (1)/ of work pressure (2)/ and deadlines to be meeting (3)/ I become restless. (4)/ No error
(5).

Ans (3): Use Passive Voice here. Hence, use ‘and deadlines to be met’.
109. He doesn’t want (1)/ to get any credit (2)/ with whatever good (3)/ he does to others. (4)/ No error (5).

Ans (3): Replace Preposition ‘with’ by ‘for’.


110. He said nothing (1)/ in response but (2)/ he clearly understood what (3)/ was being happened. (4)/ No error
(5).

Ans (4): Use of Passive Voice is wrong. Hence, use ‘was happening’.
111. The tax treaty between India and Switzerland have (1)/ been amended and we (2)/shall be able to obtain
information (3)/ about any Swiss bank account by next month. (4) /No error (5)

Ans (1): The structure of some sentences is: Noun + Preposition + Noun.
In such sentences, verb agrees with the number/person of the first subject (Noun). Here, the tax treaty is singular.
Hence, replace have by has.
112. An investor must (1)/be take into account (2)/many factors before (3)/making any financial decision. (4)/No
error (5)

Ans (2): The sentence should be in Active Voice. Hence, replace be take into account by take into account.
113. The committee will discuss (1)/ the draft in detail (2)/ and will make suggestions for the (3)/proper
implementing the scheme (4)/ No error(5)

Ans (4): Here, replace proper implementing (Gerund) the scheme by proper implementation (Noun) of the scheme.
114. A current account is a deposit account (1)/ which is offered by banks mainly (2)/ to firms and companies who
(3)/ need banking facilities very frequently. (4)/No error(5)

Ans (3): For non-living things ‘what/that’ should be used. Hence, to firms and companies which should be used.
115. Handicraft exports have an increase (1)/ in the past year (2)/ because of the innovative steps (3)/ taken by the
government. (4)/ No error (5)

Ans (1): The sentence shows event of the past. Hence, Handicraft exports increased/ Handicraft exports had an
increase should be used.
116. In his speech (1)/the Finance Minister stated (2)/ that the new tax law will be (3)/ applicable from April (1),
2010. (4) / No error (5).

Ans (3): In Indirect speech, if the reporting verb in Past Tense. T he verb of the reported speech will also be in Past
Tense. Hence, that the new tax law would be should be used.
117. Under the scheme banks (1)/ provide loans to small and medium (2)/ enterprise at two percent (3)/lower the
market rate. (4)/No error (5).

Ans (4): In Comparative Degree, that should be used. Hence, lower that the market rate will be the correct usage.
118. SEBI has recently issued (1)/show cause notices to some (2)/insurance companies seeking its explanation for
not (3)/ complying with certain norms. (4)/ No error (5)

Ans (3): Here, Companies is Plural. Hence, its possessive will be ‘their’. Hence, insurance companies seeking their
explanation(s). for not should be used.
119. I would advise you (1)/to invest in our company (2)/stock although last year (3)/ our profits decline. (4)/No
error (5)

Ans (4): Here, Past Perfect should be used. Hence, Our profits had declined will be a correct usage.
120. The next meeting can be (1)/ hold next week as (2)/ we had the first meeting (3)/ over a month ago (4)/No
error (5)

Ans (2): In Passive Voice, the structure is: Subject + to be verb + V3 (Past Participle). Hence, held (V3) next week will
be a correct usage.
121. During the reign of (1)/ the emperor of Vijayanagar, (2)/ there lived a (3)/ wise magistrate named Haripant.
(4) No error. (5)

Ans (5): No error.


122. My parents came (1)/ to town with (2)/ the intention in visiting (3)/ the University. (4) No error. (5)

Ans (2): Here, infinitive i.e. the intention to visit should be used.
123. Suman took a hundred coins (1)/ from the man and (2)/ gave him a few medicine (3)/ to make him feel better.
(4) No error. (5)

Ans (3): Here, gave him a few medicines (plural) should be used.
124. The children were (1)/ even more angrier (2)/ with the shopkeeper (3)/ for having tricked them the second
time. (4) No error. (5)

Ans (2): Here, there is an error of doubt comparatives. Hence, even more angry or even angrier
….. should be used.
125. Since it was evening by the time (1)/ the farmer reached the next village (2)/ he took shelter in the house (3)/
of a old couple. (4)/ No error. (5)

Ans (2): Here, of an old couple should be used.


126. I noticed (1)/several reporters (2)/hovering around (3)/outside of the courtroom. (4) No Error (5)

Ans (4): Here, outside the courtroom should be used.


Look at the sentences:
You can park your car outside our house.
There was nothing they could do; outside of hoping things would get better.
127. My colleague had (1)/a hard time juggling (2)/the needs of her family (3)/with the demands of her job. (4)/No
Error (5)

Ans (4): Here, and the demands of her job should be used. The word juggle (Verb) means: to try to deal with two
or more important jobs are activities at the same time.
Look at the sentences:
Working mothers are used to juggling their jobs, their children’s needs and their housework.
128. Despite all (1)/the media hype, (2)/I found the film as (3)/very disappointing. (4)/No Error (5)

Ans (3): Here, I found the film should be used. It is not proper to use ‘as’ here.
Look at the sentence:
We found the beds very comfortable. Her blood was found to contain poison.
129. Given that (1)/there was so less time, (2)/I think the students have (3)/done a good job. (4)/No Error (5)

Ans (5): No error


130. The winner of (1)/the journalist of the year award was (2)/not other than (3)/the editor’s daughter. (4)/No
Error (5)

Ans (3): Here, none other that should be used.


Look at the sentence:
Her first customer was none other that Mrs. Tendulkar.
131. The revised government’s (1)/ guidelines have reduced (2)/the number of mergers (3)/ taking place among
banks. (4)/ No error (5).

Ans (1): The groups of words ‘The revised government’s guidelines’ should be replaced by the ‘The government’s
revised guidelines’. The sentence has error of position. Clearly the governments issue revised guidelines.
132. I do not know (1)/ which of the (2)/ new trainees is (3)/ be confirmed. (4)/ No error (5).

Ans (3): The sense of the sentence demands use of Future Tense. Hence, use ‘new trainees will’ in place of ‘new
trainees is’
133. To promote India as (1)/ a tourist destination (2)/ the government has organised (3)/many cultural
programmes. (4)/ No error (5).

Ans (3): To show or regular or habitual actions, we use Simple Present. Hence, replace ‘the government has
organized’ by ‘the government organises’.
134. Banks in India (1)/cannot open ATMs (2)/except obtaining (3)/approval from RBI. (4)/No error (5).

Ans (3): Use ‘without obtaining’ for ‘except obtaining’.


135. Today the success of (1)/ companies depends on the (2)/ quality of their products and (3)/ efficient managing
staff. (4)/ No error (5).

Ans (5): No error.


136. Ashok has managed to (1)/ achieve his sales targets (2)/ for the quarter very easily (3)/ than we expected. (4)/
No error (5)

Ans (3): Here, comparison has been made. Hence, use ‘for the quarter more easily’.
137. He did not grant (1)/ their request for a loan (2)/ although it would (3)/ displease his boss. (4)/ No error (5).

Ans (3): To express rate we use article ‘a’ or pen. Hence, use ‘rupees five on every’ or ‘rupees five a/per’ or ‘rupees
five on all’.
138. At present oil companies (1)/ are making a loss of (2)/ rupees five on all (3)/ liter of petrol that they sell. (4)/
No error (5)

Ans (5): No error.


139. The manager’s belief that (1)/ Ramesh is (2)/ incompetent is not (3)/ necessary true. (4)/No error (5)

Ans (4): The groups of words ‘necessary true’ should be replaced by necessarily true’. The word ‘necessarily’ is an
Adverb that modifies the Adjective ‘true’.
140. Since I was doing (1)/ my MBA I initially (2)/ chose to specialise (3)/ in marketing management. (4)/ No error
(5).

Ans (1): Use ‘when I was doing’ for ‘since I was doing’ according to the sense of the sentence.
141. Taking regular breaks (1)/ from work are (2)/ important for health (3)/ and reduces stress. (4)/ No error (5)

Ans (1): Here, Gerund ‘taking’ has been used as subject that is Singular. Hence, it will take Singular verb. Hence,
Taking regular breaks from work is … will be correct sentence.
142. Children of smokers (1)/ are more affected by (2)/ diseases like asthma (3)/ and tuberculosis. (4)/ No error (5)

Ans (5): No error.


143. We often greet (1)/ one person by (2)/ joining our hands (3)/ and saying Namaste. (4)/ No error (5)

Ans (2): Replace group of words ‘one person by’ with ‘a person by’. Article ‘a’ generalizes the sense, while the word
‘one’ specializes.
144. The horse who (1)/ won the race (2)/ has been removed (3)/ from the stable. (4)/ No error (5)

Ans (1): The horse which …. Will be correct sentence.


145. The traffic signals (1)/ at the junction (2)/ did not worked (3)/ properly in the morning. (4)/ No error (5)

Ans (3): ‘Did not’ is followed by V1 from (plural) of a Verb. Hence, the traffic signals at the junction did not work …
will be correct sentence.
146. The child weight (1) less than what (2)/ is expected (3)/ at his age. (4)/ No error (5)

Ans (1): The word weight is a Noun. Its verbal form is weigh. Hence, the child weighs (Verb) …. Will be correct
sentence.
147. The cup of tea (1)/ which was kept (2)/ upon the table (3)/ fell down. (4)/ No error (5)

Ans (5): No error.


148. The pass percentage has (1)/ gone up by twenty per cent (2)/ from what used to (3)/ a decade earlier. (4)/ No
error (5)

Ans (3): Replace group of words ‘from what used to’ by ‘from what used to be’.
149. There is a (1)/ lot of schemes (2)/ for senior citizens (3)/ to invest money. (4)/ No error (5)

Ans (1): Here ‘a lot of schemes’ is in plural form. Hence, there are lot of schemes … will be a correct sentence.
150. He was a (1)/ miser son who (2)/ never spent a penny (3)/ on his own children. (4)/ No error (5) Ans (2): He
was a miser parent/father … will be a correct sentence. The word ‘son’ has been wrongly used.

151. The basketball match (1)/ was organized and (2)/ the sponsored by his (3)/ father’s company. (4)/ No Error
(5).

Ans (3): It is improper to use an article before a verb. Hence, sponsored by his should be used here.
152. There were many (1)/ factors that contributed (2)/ to the success (3)/ of this experiments. (4)/ No Error (5).

Ans (4): Here of this experiment/of these experiments should be used.


153. Child labour is (1)/ considered to be (2)/ illegal in (3)/ many countries. (4)/ No Error (5).

Ans (2): Here, it is improper to use ‘to be’ (Probability). Hence considered should beused.
154. She always remembers (1)/ to switch off all the (2)/ lights and fans before (3)/ leaving a rooms. (4)/ No Error
(5).

Ans (4): Here, leaving rooms/ leaving a room should be used.


155. Inspite of the (1)/ heavy raining Raj (2)/ decided to go (3)/ for the meeting. (4) / No Error (5) No Error (5).

Ans (2): Here, Noun i.e. rain should be used. Hence, heavy rain Raj should be used.
156. Ragi may not taste (1) /good but it has (2) / very high (3) / nutrition value. (4) /No Error (5).

Ans (3): Here, Article ‘a’ i.e. a very high should be used.
157. She wanted to reached (1) /home as early (2) / as possible because (3) / it was getting dark. (4) / No Error (5)

Ans (1): The full form of an Infinities is : to +V1 (Plural). Hence, She wanted to reach should be used.
158. Rohan’s happiness knew no (1) / bounds when the results (2) / were announce because (3) / he had won the
competition. (4) / No Error.

Ans (3): Structure of Past Simple (Passive Voice) is: Subject + was/were + V3 (Past Participle). Hence, were
announced because should be used.
159. The royal guards were instruction (1) / to keep the palace (2) / gates closed as the (3) / villagers were agitated.
(4) / No Error (5).

Ans (1): Here, the royal guards were instructed should be used.
160. They were hoping (1) / to reach of time (2) / for the lecture (3) / but they were late. (4) / No Error.

Ans (2): Idiom on time / in time means: not late. Hence, to reach on time should be used.
161. The main objective of (1) / the committee members are (2) / to recommend reforms (3) / to the education
ministry. (4) / No error (5)

Ans (2): The structure of some sentence is : Noun + Preposition + Noun. For example,
The main objective of the committee…
In such sentences, the Verb is used according to the nimber and Person of the Noun used before the preposition.
Hence
The main objective of the commit

Singular
Tee members is … will be the

Singular Correct sentence.
162. The seminar was attended (1) / by investment consultants (2) / who discussed the risks (3) / of investing
abroad. (4) / No error. (5)

Ans (4): Replace ‘of investing abroad’ by ‘of investment abroad’.


163. The country’s constitution (1) / came into force (2) / three years since it (3) / gained its independence. (4) No
error. (5)

Ans (3): According to the sentence, replace ‘three years since it’ by ‘three years after it’.
164. Over eight million (1) / people have fallen (2) / below the poverty line (3)/ owing to rising in flation. (4) / No
error (5)

Ans (5): No error.


165. The governments has decided to (1) / dedicate more than fifteen crores of rupees (2) / of the research and
development (3) / of ecofriendly sources of energy. (4) / No error. (5)

Ans (3): The word ‘dedicate’ will take Preposition ‘to’. Hence ….to the research and development’ …. Will be
correct usage.
166. Over half of the (1) / domestic flights were (2) / delayed because (3) / of bad weather conditions.
(4) / No error. (5)

Ans (4): It will be proper to use ‘of bad weather condition’. Here, the word ‘condition’ is superfluous
167. Banks have being asked (1) / to prepare a list of (2) / beneficiaries for the scheme (3) / as per the government’s
guidelines. (4) / No error. (5)

Ans (1): The structure of sentences in Passive voice of Present Perfect is Subject + have been/has been + V3. Hence,
replace ‘Banks have been asked’ by ‘Banks have been asked’.
168. It rained all (1) / day last week so (2) / we did not (3) / enjoy our vacation. (4) / No error. (5)

Ans (1): It rained everyday last week … will be correct usage.


169. The manager was pleased (1) / with Anand’s report (2) / as it was (3) / too detailed and systematic. (4) No
error. (5)

Ans (4): It is a better to use ‘detailed and systematic’. When ‘too’ is used in some sentences, it Express of the quality
and it shows a vice instead of virtue.
170. Many families in the district (1) / have sold partly of their land (2)/ in order to pay (3) / for their children’s
education. (4) No error. (5)

Ans (2): Use of Adverb ‘partly’ is wrong. Hence, ‘have sold part of their land’ …. Will be correct usage.
171. Rajiv is the best person (1)/ to advise you since (2)/ he faced plenty of difficulty (3)/ while setting up his
business. (4)/ No error (5)

Ans (3): ‘Plenty of’ is followed by Plural Noun. Hence, replace group of words ‘he faced plenty of difficulty’ by ‘he
faced plenty of difficulties’.
172. The University that (1)/ I taught was (2)/ located on the (3)/ outskirts of the city. (4)/ No error (5)

Ans (1): To provide clarity to the meaning of the sentence, it is proper to use ‘the University where’ in place of ‘the
University that’.
173. Most of my (1)/ co-workers have been (2)/ transferred to various (3)/ neighbour districts. (4) / No error (5)

Ans (4): To qualify a Noun, an Adjective is used. Hence, use ‘neighbourly districts’ in place of ‘neighbour districts’.
174. Though my father spent (1) / a lot of money on (2)/ our education he refused to (3)/ give us no pocket money.
(4)/ No error (5)

Ans (2): The word ‘refuse’ gives a negative sense. Hence, replace ‘give us no pocket money’ by ‘give us any pocket
money’.
175. Often ideas and concepts (1) / are passed on from (2)/ one generation to the next (3) / without being
questioned. (4)/ No error (5)

Ans (5): No error.


176. Funds that are collected (1) / in the morning has (2)/to be disbursed as (3)/ new loans by afternoon. (4)/ No
error (5).

Ans (2): The subject of the sentence, ‘funds’ is in Plural Number. Hence, replace ‘in the morning has’ by ‘in the
morning have’.
177. Your promotion depends (1)/ on the number of (2)/ projects your handling (3) / during the year.
(4) / No error (5).

Ans (3): Use ‘projects you handle’ in place of ‘projects your handling’.
178. Our team has spent (1)/ the night entirely (2)/ awake trying to decide (3)/ what strategy to adopt. (4)/ No
error (5)

Ans (3): Replace ‘awake (V1) trying to decide’ by ‘awaken (V3) trying to decide’.
179. I have finally managed (1)/ to convince my parents (2)/ to allow myself (3)/ to study abroad. (4)/ No error (5).

Ans (5): No error.


180. When he took a loan (1)/ for the first time (2)/he was so nervous that (3)/ he repay it early. (4) / No error (5).

Ans (4): The sense of the sentence shows past. Hence, use ‘he repaid it early’ in place of ‘he repay it early’.
181. People who intend (1)/ to visit the tourist spots (2)/ are always thrilling (3)/ to see the scenario here. (4)/ No
Error (5).

Ans (3): Here, V4 i.e. thrilling should be replaced by thrilled (Adjective).


182. In such delicate matters, (1)/ we often go with (2)/ his advice as he has (3)/ been handling such cases
effectively. (4)/ No Error (5).

Ans (5): No Error.


183. You should thing that (1)/ of all the possibilities (2)/ before you take (3)/ any decision. (4)/ No Error (5).

Ans (1): The use of ‘that’ is superfluous.


184. He was too tired that (1)/ he could not cross (2)/ the street even with (3)/ the help of a porter. (4)/ No Error
(5).

Ans (1): Here, too should be replaced by so.


Look at the sentences:
He was too weak to walk.
He was so weak that he couldn’t a\walk.
185. My desire to (1)/ meet the President (2)/ without prior (3)/ appointment. (4)/ No Error (5).

Ans (1): Replace My desire to by My desire is to or I desire.


186. Whenever a man attain fame, (1)/ his personal qualities are (2)/ imitated by others who (3)/ are close to him.
(4)/ No Error (5).

Ans (1): Singular subject agrees with singular verb. Hence, whenever a man attains fame will be a correct usage.
187. Rivers, mountains and deep forests (1)/ are the places (2)/ mostly like by (3)/ people living in urban areas.
(4)/ No Error (5).

Ans (3): Replace mostly like by by liked most by.


188. When we visited his office (1)/ we found that (2)/ he was sipping coffee (3)/ with some of his colleagues. (4)/
No Error (5).

Ans (5): No Error.


189. For giving up (1)/ the bad habit of smoking, (2)/ use of chewing gum or (3)/ similar other method can be
helped. (4)/ No Error (5).

Ans (4): Here, similar other method can be helpful/useful (Adjective) should be used.
190. His obviously reluctance (1)/ was viewed seriously by (2)/ his superiors and (3) he was suspended. (4)/ No
Error (5).

Ans (1): Here, His obviously (Adverb) reluctance should be replaced by His obvious (Adjective) reluctance
because an Adjective qualifies a Noun.
191. Which sectors (1)/ have been affected (2)/ the most of (3)/ the global recession? (4)/ No error (5).

Ans (3): Replace ‘the most of’ by ‘the most by/the most because of’.
192. One of the (1)/ executives who work (2)/ at the head office (3)/ has confirmed the news (4)/ No error (5).

Ans (5): No error.


193. It took myself (1)/ four months to (2)/ convince them to (3)/ take this step. (4)/ No error (5).

Ans (1): Replace ‘It took myself’ by ‘It took me’.


Look at the sentences: I myself do my work. He took me away..
194. Anita’s new office (1)/ is quite spacious but (2)/ most of the furniture is (3)/ old and should be changed. (4)/
No error (5).

Ans (5): No Error.


195. According to the terms (1)/ of the agreement we have to (2)/ pay five percent commissions (3)/ to the travel
agent. (4)/ No error (5).

Ans (3): Replace ‘pay five percent commission’ by ‘pay five percent commission’. It is incorrect to use ‘commissions’
196. Unless you work out all (1)/ the details and get the necessary (2)/ approvals they will not want (3)/
investment in the project. (4)/ No error (5).

Ans (3): Replace ‘approvals they will not want’ by ‘approvals they will be not allow/facilitate’.
197. Hearing the tone of (1)/ my colleague’s voice I (2)/ realised that he was (3)/ upsetting about something (4)/
No error (5).

Ans (4): Replace ‘upsetting about something’ by ‘upset about something’. It is proper to use Adjective (upset) here.
198. Being recently arrived (1)/ in the city we (2)/ visited our neighbours to (3)/ get to know them. (4)/ No error
(5).

Ans (1): ‘Being recently arrived’ should replaced by ‘Having recently arrived’. It is proper to use past participle
from here.
199. Since I had ordered (1)/ a large number of (2)/ books the salesman agreed (3)/ to give me a discount (4)/ No
error (5).

Ans (1): Preposition ‘for’ is missing here. Hence, replace ‘since I had ordered’ by ‘Since I had ordered for’.
200. We have not yet (1)/ reached a stage that (2)/ we can think of (3)/ expanding our business overseas. (4)/ No
error (5).

Ans (5): No error.


201. When we (1)/reached the shops, (2)/we find that (3)/they were all closed.(4)/No Error (5)

Ans (3): The sentence shows past time. Hence, we found that (Past Simple) should be used here.
202. Poor people (1)/has no money (2)/therefore they cannot afford (3)/proper medical facilities. (4)/No Error (5)

Ans (2): Here, the subject (poor people) is Plural. Hence, have no money (Plural Verb) should be used.
203. The course is for (1)/ something who is interested (2)/in learning (3)/about computers. (4)/No Error (5)

Ans (2): Here, someone who is interested …. Should be used.


204. We were surprised (1)/that she participated (2)/at the performance (3)/held at NCPA. (4)/No Error (5)

Ans (3): Here, in the performances …. Should be used.


Look at the sentence:: He didn’t participate in the discussion
205. As soon as (1)/I getting my (2)/first salary (3)/I spent all my money. (4)/No Error (5)

Ans (2): Here, Past Simple i.e. I got my … should be used.


206. The principal (1)/introducing me (2)/to some of her (3)/teaching staff. (4)/No Error (5)

Ans (2): Here, Past Simple i.e. introduced me … should be used.


207. The doctors said that (1)/he was making good progress (2)/and would not (3)/need an operation. (4)/No
Error (5)

Ans (5): No error.


208. Instead of (1)/to buy books, (2)/I borrow them (3)/from the library. (4)/No Error (5)

Ans (2): Here, Gerund i.e. buying books … should be used.


209. By the times (1)/we reached the classroom, (2)/the lecture had (3)/already begun. (4)/No Error (5)

Ans (1): Here, By the time … should be used.


210. The Indian government (1)/should make sure that (2)/men and women are (3)/given equal opportunities.
(4)/No Error (5)

Ans (5): No error.


211. If you want to (1)/keep good health (2)/you must eats (3)/a lot of vegetables. (4)/No Error (5) Ans (3): Modals
should, would, must etc. agree with V1 (original form of verb). Hence, you must eat …. Should be used.

212. We were help (1)/to carry (2)/the piano upstairs (3)/by one of our neighbours. (4)/No error (5) Ans (1): Look
at the structure of a sentence in Passive Voice in Past Simple. Subject + was /were + V3 (Past Participle). Hence, we
were helped …. Should be used here.

213. We should all try (1)/to help the police (2)/in their efforts (3)/to combating crime. (4)/No error (5)

Ans (4): Here, to combat crime (infinitive = to + V1) should be used.


214. My name is Rahul (1)/and I am coming (2)/from a beautiful state (3)/of India. (4)/ No error (5) Ans (2): To
express simple or permanent activity, we use Present Simple. Hence and I come …. Should be used.

215. In recent months (1)/large number of equipments (2)/has been steal (3)/from construction sites.
(4) /No Error (5)

Ans (3): Here, Subject i.e. a large number of equipments is plural. Moreover, in Passive V3 (Past Participle) should
be used. Hence, have been stolen …. Should be used here.
216. I forget (1)/to pack a set of (2)/clean clothes (3)/for the picnic. (4)/No Error (5).

Ans (1): Here, Present Perfect i.e. I forgotten …. Should be used. We can also use Past Simple.
217. It is important (1)/to be looking (2)/both ways before (3)/crossing the road. (4)/No Error (5)

Ans (2): Here, to look …. Should be used.


218. The Indian Government’s (1)/primary goal (2)/is to reduction (3)/of poverty. (4)/No Error (5)

Ans (3): Here, use of ‘to’ is superfluous. Hence, is the reduction …. Should be used.
219. I paid a lot of money (1)/for the camera (2) /and I expects it (3) /to work. (4) No Error (5)

Ans (3): Here, and I expect/expected it should be used.


220. Some peoples who (1)/live in the Netherlands (2)/cycle to work (3)/and back daily. (4) No Error (5)

Ans (1): Here, Some people who … should be used. Peoples means nation.
221. Most of the popular tele-serials (1)/ are not only illogical (2)/ in their story line (3)/ but also crude in their
presentation. (4)/ No error (5).

Ans (5): No error.


222. I am trying to convince him (1)/ for the last two days to come (2)/ and live with me (3)/ till his father’s anger
cools down. (4)/ No error (5).

Ans (1): The Present Perfect Continuous is used for an action which began at some time in the past and is still
continuing; as, He has been sleeping for five hours (and is still sleeping).
Therefore, ‘I am trying’ should be replaced with ‘I have been trying’
223. It is a pity that a son (1)/ born from very good parents (2)/ should live a life of (3)/ misery and deprivation of
the worst order. (4)/ No error (5).

Ans (3): The Present Continuous is used for an action going on at the time of speaking; as. She is singing (now).
Therefore ‘should live’ should be replaced with ‘is living’.
224. I would have asked him (1)/ to leave our house immediately (2)/ if my father would not have been (3)/ at
home and awake. (4)/ No error (5).

Ans (3): The given sentence expresses a past possibility that remained a possibility. The structure will be:
If + Sub-ordinate clause [Simple Past/Past Perfect] + Principal clause [Past Perfect] For example,
If he had worked hard, he would have passed.
If he had been Prime Minister. I would have abolish examinations. If father were at home. I would have got the
money easily.
Therefore, ‘If my father would not have been’ should be replaced with ‘If my father were not’.
225. Mala’s two brothers Rupesh and Bhupat (1)/ are very intelligent and hardworking (2)/ but I am sure (3)/ I like
the latter most. (4)/ No error (5).

Ans (4): ‘I’ comes for Mala. Therefore ‘She likes’ will be correct usage.
226. I am grateful to you (1)/ and all your friends (2)/ for the showed sympathy (3)/ and kindness towards me (4)/
No error (5)

Ans (4): The word Kindness is followed by Preposition ‘to;. For example, I can never repay her many kindness to
me.
227. While he was (1)/ walking along the road (2)/ a speeding car (3)/ knocked down to him (4)/ No error (5)

Ans (4): The phrase knock somebody down and make them fall to the ground. For example, He was knocked
down. Hence, the correct down.
Hence, the correct sentence will be:
While he was walking along the road, a speeding care knocked him down.
228. Though none of his (1)/ so-called well – wishers forwarded to help (2)/ I helped him by (3)/ completing his
work on time (4)/ No error (5)

Ans (2): The correct sentence will be: Though none of his so – called well-wishers came forward to help, I helped
him by completing his work on time.
229. From the way he was talking (1)/ It was clear (2)/ that he had (3)/ no control on himself. (4)/ No error (5)

Ans (5): No error.


230. Your over-dependent on (1)/ others even for (2)/ trivial matters may (3)/ prove disadvantageous (4)/ No error
(5)

Ans (1): Your over dependence on is the correct usage. As dependent is an Adjective, its Noun
form will be used as a Subject.
231. People who are (1)/ fortunate enough to own (2)/ a personal library are always (3)/ held in high esteem by
me. (4)/ No error (5)

Ans (5): No error.


232. A small loaf of bread (1)/ given with affection (2)/ is far superior to (3)/ a delicious dish served with
indifferently. (4)/ No error (5)

Ans (4): The word indifferently has been wrongly used. It should be replaced with its Noun form indifference.
Well knowingly, an Adverb modifies a Verb.
233. The incentive seems to (1)/ having been worked well (2)/ because the number of employees going on leave
(3)/ has reduced significantly. (4)/ No error (5)

Ans (2): The correct sentence will be: The incentive seems to have worked well ….
234. His dishonest acts have (1)/ made his parents (2)/ bent their heads (3)/ in shame. (4)/ No error (5)

Ans (3): The structure of the sentence with causative Verbs will be Subject + have/has + made + object +V1.
For example: His actions made him laugh. Hence, bent should be replaced with bend.
235. Everybody know (1)/ that his failure can (2)/ be attributed only (3)/ to his lack of practice. (4)/No error (5)

Ans (1): Everybody is a Singular Verb. Hence the correct sentence will be: Everybody knows …..
236. In despite of his (1)/ being the small boy (2)/he offered to (3)/fight the giant. (4)/No error (5).

Ans (1): Inspite of or despite is the correct usage.


237. He was delighted (1)/by the intelligent and brightness (2) / of the scholars (3) / who used to visit him. (4)/ No
error (5)

Ans (2): Intelligent Adjective should be replaced with intelligence (Noun).


238. We were happy that (1) / the audience responded well (2) / and gave all the speakers (3) / a patiently
listening. (4)/ No error (5)

Ans (4): ‘A patiently listening’ should be replaced with ‘a patient listening’, because listening is a Noun and to
qualify a Noun, an Adjective is used.
239. He received timely support (1) / from his elder brother (2) / who is working abroad (3) / for the last six years.
(4) / No error (5)

Ans (3): ‘Who is working’ should be replaced with ‘who has been working’ because the sentence indicates the sense
of period of time.
240. The notorious gang opened (1) / the door quietly and (2) / escaped in the dark with (3) / whatever they would
collect. (4) / No error (5)

Ans (4): ‘Whatever they would collect’ should be replaced with ‘whatever they had collected’.
241. One of the security men (1) / rushed forward and asked (2) / me whether I had (3) / anything objectionable.
(4) / No error (5)

Ans (5): No error.


242. We could not (1) / believe that one (2) / of us was (3) / responsible with the act. (4) / No error (5)

Ans (4): ‘Responsible with the act’ should be replaced with ‘responsible for the act’.
243. We are now (1) / reliably learnt that (2)/he was involved (3)/ in the bank robbery. (4)/No error (5)

Ans (5): No error.


244. Now a days the cost of living (1)/ is so high that (2)/ people find it difficult (3)/to make both ends meeting.
(4)/No error (5)

Ans (4): ‘To make both ends meet’ is the correct form of phrase.
245. Karnavati is (1)/ one of the leading (2)/ business centres (3)/ in our state. (4)/ N error (5)

Ans (5): No error.


246. It was clear from the way (1)/they were behaving (2)/that they had been (3)/lost their senses

.(4)/ No error (5)


Ans (3): Past Perfect Tense will be used. The structure is: Subject + had + past participle
247. That boy possess (1)/three beautiful pens (2)/but he would not (3)/show them to any one. (4)/ No error (5)

Ans (1): ‘That boy possesses’ is the correct usage, as the verb must agree with its subject in number and person.
248. A small piece (1)/of bread is (2)/better than (3)/having nothing to eat. (4)/ No error (5)

Ans (5): No error.


249. Though I had been (1)/his friend for quite a long time (2)/I refused to help him (3)/because his ill nature. (4)/
No error (5)

Ans (4): ‘Because of’ is the correct idiomatic form.


250. When he had been (1)/walked along the road (2)/a wild and ferocious dog (3)/hit him hard and knocked him
down. (4)/ No error (5)

Ans (2): The structure of Past Perfect Continuous is: Subject + had been + verb + ing
251. I am grateful to you (1)/and all your friends (2)/for showing sympathy (3)/and kindness with me (4)/ No
error (5)

Ans (4): The preposition ‘with’ should be replaced by ‘to’.


252. The leader was so shrewd (1)/that he could not deceive (2)/by the words of (3)/the sycophant courtiers. (4)/
No error (5)

Ans (2): Passive voice will be used. i.e. ‘That he could not be deceived’, is the correct form.
253. He is the man (1)/who I know (2)/has helped my son (3)/in the final examination. (4)/No error (5)

Ans (3): Simple Past is used for an action happened in the past and showing past time.
254. I have been (1)/living in, Hyderabad (2)/at my uncle (3)/since my birth. (4)/ No error (5)

Ans (3): The preposition ‘at’ should be replaced by ‘with’.


255. Every one of us know (1)/that he is not capable of (2)/remaining under water (3)/for such a long time. (4)/ No
error (5)

Ans (1): Singular subject agrees with singular verb.


256. Your television set (1)/is superior to (2)/our television set (3)/by all respects. (4)/ No error (5)

Ans (4): ‘In all respects’ is the correct idiomatic form.


257. The boy who was guilt for (1)/having broken the window glass (2)/came out (3)/with the truth. (4)/ No error
(5)

Ans (1): ‘The boy who was guilty of’ is the correct usage.
258. Ramesh has been both (1)/a dishonestly person (2)/and a gambler (3)/since his childhood. (4)/ No error (5).

Ans (2): ‘Dishonestly’ should be replaced by adjective ‘dishonest’.


259. The number of employees (1)/reporting sick (2)/has reduced significantly (3)/because of the incentive. (4)/ No
error (5).

Ans (2): ‘Reporting’ should be replaced by ‘reported’, as it adjective.


260. Sunil refused to admit that (1)/he did not know (2)/ what to operate (3)/the new machine. (4)/No error (5)

Ans (3): The group of words ‘what to operate’ should be replaced by ‘how to operate.
Look at the following sentences:
I know how to swim. I know what to do.
261. They waited patient (1) / for the chairman’s speech (2)/to end in order (3)/to begin the discussion. (4)/No
error (5)

Ans (1): The group of words ‘they waited patient’ should be replaced by ‘they waited patiently’. We use an
‘Adverb’ to modify a verb. The Adverb form of the word ‘patient’ is patiently.
262. On his next (1)/official visit to Delhi (2)/the plans to (3)/visit his old colleagues. (4)/No error (5)

Ans (5): No error.


263. He has refused (1)/not to take (2)/ the promotion although (3)/ he is eligible. (4)/ No error (5) Ans (2): Here,
the word ‘refuse’ has a negative sense. Hence, ‘not to take’ should be replaced by ‘to take’.

264. We were so late (1) / than the meeting (2) / was almost over (3)/when we arrived. (4) / No error (5)

Ans (2): ‘So …. That’ is the correct correlative. Hence, ‘than’ should be replaced by ‘that’.
265. All this document (1)/have been destroyed (2)/ in the fire (3)/which occurred last month. (4) / No error (5)

Ans (1): Here, the auxiliary verb of the sentence is plural. Hence, ‘all this document’ should be replaced by ‘all these
documents’.
266. Mr. Gupta insisted (1)/ that he was (2)/ fully prepare (3) / to present the proposal. (4) / No error (5)

Ans (3): Passive Voice takes Past Participle form of the verb. Hence, ’fully prepare’ should be replaced by ‘fully
prepared’.
267. Arjun was (1) / not selected (2) / for the post (3) / Inspite his hard work. (4) / no error (5)

Ans (4): ‘Inspite’ should be replaced by ‘inspite of’. We can use ‘despite’ too.
Look at these sentences:
Inspite of his age he still leads an active life.
They went swimming inspite of all the danger signs.
Despite applying for hundreds of jobs, he is still out of work.
268. The decision to open (1)/ offices in Shimla (2) / were taken at (3)/ the last Board meeting. (4) / No error (5)

Ans (3): Hence, the subject of the sentence is ‘decision’ which is in Singular Number. Hence, Singular Verb should
be used. Hence, replaced ‘were’ by ‘was.
269. There has been (1)/a fall from (2) / the profits of (3) / our company recently. (4) / No error (5)

Ans (2): Replace ‘a fall from’ by ‘a fall in’.


270. Hostility between the (1)/two groups have (2)/increased in the (3)/past few months. (4)/No error (5)

Ans (2): The subject of the sentence ‘Hostility’ is singular. Hence, Hostility between the two groups has … will be a
correct sentence.
271. Her class is very special (1)/because it has children (2)/with many different (3)/abilities and skills. (4)/No
error (5)

Ans (3): Replace group of words ‘with many different’ by ‘with different/ of different’.
272. Many peoples were (1)/brought to safety (2)/by the army helicopters (3)/from the flood hit area. (4)/No error
(5)

Ans (1): The word ‘people’ is a plural Noun. Hence, ‘Many people were’ will be a correct usage.
273. It is difficult to (1)/ understand the problems (2)/that the physically challenged people (3)/encounters in their
daily life. (4)/No error(5)

Ans (4): The subject of the clause ‘the physically challenged people’ is plural. Hence, ‘encounter in their daily life’
should be used because plural subject agrees with plural verb.
274. I had gone only a little way (1)/down the street (2)/when I realised that (3)/I had not lock the door. (4)/No
error(5)

Ans (4): The structure of Past Perfect Tense is: Subject + had +V3 (Past Participle).
Hence, ‘I had not locked the door’ should be used.
275. Mammoths were a (1)/species of elephants (2)/who lived millions of years ago (3)/but are now extinct. (4)/No
error(5)

Ans (3): Replace group of words ‘who lived millions of years ago’ by ‘which lived millions of years ago’.
276. The sink is (1)/so dirty that (2)/one needs a very strong (3)/abrasive to cleaning it. (4)/No error (5)

Ans (4): Replace group of words ‘abrasive to cleaning it’ by ‘abrasive to clean it’ (Purpose).
277. The talks ended (1)/abruptly when one of (2)/the delegate walked (3)/out in protest. (4)/No error (5)

Ans (3): ‘One of’ is followed by ‘Plural Noun/Pronoun’. Hence, ‘the delegate walked’ should replaced by ‘the
delegates walked’.
278. A twenty year old (1)/absconder was caught (2)/in a local restaurant (3)/this morning. (4)/No error (5)

Ans (5): No error.


279. If she will secure more than (1)/90% marks in the exams (2)/I will give her (3)/a suitable reward. (4)/No error
(5)

Ans (1): When two events are likely in future, if is used before the sentence used in Simple Present. Hence, replace
‘if she will secure more than’ by ‘if she secures more than’
280. The father divided (1)/the property equally (2) /between his son(3) /and daughter. (4)/ No error (5)

Ans (5): No error


281. I am thinking (1) /you were aware (2) /of the rules and regulation (3) /before joining. (4) /No error (5)

Ans (1): The word think is a stative verb. It should not be used in Present Progressive. Hence, I think should be
used here.
282. I knows a very good doctor (1)/ in my village (2)/who will be able to (3)/ cure your disease. (4)/ No error (5)

Ans (1): I know ….. is a correct usage.


283. I am not scared (1) / to be travelling (2) /in the dark (3) / alone at night. (4) /No error (5)

Ans (2): Scare is followed by Preposition ‘of’. Hence, of travelling ….should be used.
284. The elevator in the building (1) / is under repair, (2) /we regret for (3) / the inconvenience caused. (4) / No
error (5)

Ans (5): No Error.


285. The bus waiting (1) /for all the passengers (2) /to enter (3) /before it took off. (4) /No Error (5) Ans (1): If two
events happen in the past, the event is expressed in Past Perfect. Hence, the bus had waited …. Should be used here.

286. Daily prayer and meditation (1) /is the keys (2) /to a peaceful (3) /and happy life.(4)/No Error (5)

Ans (2): Here the subject is plural. Hence, are the key(s) should be used.
287. The police making (1) /everyone leave (2) /the building premises (3) /immediately. (4) /No Error (5)

Ans (1): Here, Causative Verb form i.e. The police made should be used.
Look at the sentences I made him laugh. He made her weep.
288. Mother and I (1) /generally go to the store (2) /and pickup food items (3) /for the charitable.(4)/No Error (5)

Ans (4): Here, for the charity (Noun) should be used.


289. The new project is (1)/ to big for (2)/ the trainee to handle (3)/ on his own. (4)/ No Error (5).

Ans (2): In some sentences, too +Adjective / Adverb + Infinitive is used. In such sentences, too + Adjective/Adverb
shows cause and the Infinitive shows result.
Look at the sentences: He is too weak to run.
She is too slow to win the race. Hence, the new project is too big for …. Is correct sentence.
290. The explanation that (1)/ he gave for (2)/ missing the meeting (3)/ were not satisfactory. (4)/ No Error (5)

Ans (4): The subject of the sentence is ‘the explanation’ which is singular. According to the rule, S- S, P-P, it will take
Singular Verb.
Hence, replace ‘were not satisfactory’ by ….’Was not satisfactory’.
291. Mr. Singh’s son has applied (1)/ to the post (2)/ of Assistant Manager (3)/ in a construction company. (4)/ No
Error (5)

Ans (2): The word apply takes the preposition ‘for’. Hence, Mr. singh’s son has applied for … is the correct
sentence.
292. The manager has sent (1)/ many reminders to Mr. Rao (2)/ to repay the loan but (3)/ has not received to reply.
(4)/ No Error (5)

Ans (4): Use of double negatives is improper. Hence, …. But has not received any reply ….. is the correct sentence.
293. Beside the Chairman (1)/ all the Committee members (2)/ were present (3)/ at the shareholders meeting. (4)/
No Error (5)

Ans (1): The word ‘beside’ should be replaced by ‘besides’ means: In addition to somebody/something.
Look at the sentence: We have lots of things in common besides music.
294. He submitted (1)/ the request for compensation (2)/ of time but (3)/ it was denied. (4)/ No Error (5)

Ans (3): Use Preposition ‘for’ in place of ‘of’.


295. Unfortunately today many (1) / parents cannot afford (2) / to sent its (3) / children to school. (4)

/ No Error (5)
Ans (3): The subject of the sentence ‘parents’ is in Plural Number. Hence, use ‘their’ in place of ‘its’.
296. He was convinced (1)/ that he (2)/ loss the account (3)/ because of bad luck. (4)/ No Error (5) Ans (3): As the
reporting Verb of Indirect Speech is in Past Tense, the Reported Speech will also be in Past Tense. Hence, ….. he had
lost the account because of bad luck will be correct sentence.

297. Many customers have (1)/ complained because (2)/ his goods was (3)/ inferior in quality. (4)/ No Error (5)

Ans (3): As the word ‘goods’ is in Plural Verb. Hence, use ‘were’ in place of ‘was’.
298. In his opinion (1)/ every senior citizens (2)/ will benefit (3)/ form the new rule. (4)/ No Error (5)

Ans (2): Every senior citizen will be correct usage. Every takes Singular Noun/Pronoun.
299. It is sad that in (1)/our country education is (2)/ available only to those (3)/ whom can afford it (4)/ No error
(5)

Ans (4): ‘Who’ is used to show which person or people you mean’. ‘whom’ is used instead of ‘who’ as an object of a
Verb or Preposition.
Look at the sentences:
The people who called yesterday want to buy the house. To whom should I write?
Whom did they invite?
Hence, ‘whom can afford it’ should be replaced by ‘who can afford it’.
300. I was determined (1)/ to learn everything (2)/ I could from the people (3)/ off the village. (4)/ No error (5).

Ans (4): Replace group of words ‘off the village’ by ;of the village’ The Preposition ‘off shows separation while ‘ of
shows relation or possession.
301. During the holidays (1)/ I am often worked (2)/ in my uncle’s shop to (3)/ earn some pocket money. (4)/ No
error (5).

Ans (2): It is proper to use /Simple Present (Active) here. Hence, replace ‘I am often worked’ by ‘I often work’.
302. Most banks did not taken (1)/ her plan to set up (2)/ a factory in (3)/ the areas seriously. (4)/ No error. (5)

Ans (1): Simple Past (Negative): Subject + did not + V1. Hence, Most banks did not take ……will be a correct
sentence.
303. Every year we have (1)/ a function where (2)/ the children of our staff (3)/ is awarded scholarships. (4)/ No
error (5).

Ans (4): Here the subject ‘Children’ is in Plural Number. Hence, it is proper to use ‘are awarded scholarships’
instead of ‘is awarded scholarships’.
304. Many of the funds (1)/ I needed to buy (2)/ my own house were (3)/ given by my father. (4)/ No error (5).

Ans (1): Replace ‘Many of the funds’ by ‘Most of the funds’.


305. Since he was from (1)/ the district and able to (2)/ speak the language locally, (3)/ they trusted him (4)/ No
error (5).

Ans (5): No error.


306. Every month he would (1)/ meet the employees to (2)/ discuss their problems (3)/ and answer their question.
(4) / No error (5)

Ans (5): No error.


307. I had to work full time (1)/ so I could not devote (2)/ as much time as (3)/ I wanted to playing cricket. (4)/ No
error.

Ans (3): The word ‘locally’ is an Adverb that modifies a Verb. It is proper to use ‘speak the local language’.
308. He did not even (1)/ have no property (2)/ to sell to (3)/ cover his losses. (4)/ No error (5).

Ans (2): Replace ‘have no property’ by ‘have any property’. Double negatives make wrong sense here.
309. Over a thousand tourist (1)/from all over the world (2)/come to visit the (3)/national park every week. (4)/No
error. (5)

Ans (1): Replace groups of words ‘over a thousand tourist’ by ‘over a thousand tourists’. A Plural Numeral
Adjective is followed by a Plural subject or Noun.
310. It will sooner (1)/be compulsory for all (2)/the employees to follow (3)/these new guidelines. (4)/No Error.(5)

Ans (5): No Error.


311. The bank is (1)/planning to hire (2) over eight hundred officers (3)/on a contractual basic. (4)/No Error. (5)

Ans (4): Replace a group of words ‘on a contractual basic’ by ‘an a contractual basic’. The use of
basic is improper here.
312. In his opinion (1)/have a good education (2)/is the way (3)/to achieve success. (4)/No error. (5)

Ans (2): Use ‘to’ before group of words’ have a good education’. Infinitive is used as a subject too.
For example, To err is human.
313. If we want to avoid (1)/making a loss we (2)/have no option unless (3)/to raise our prices. (4)/No Error (5)

Ans (3): The use of connective ‘unless’ is improper. It should be substituted by connective ‘but’.
314. The project aims (1)/at developing India (2)/small and medium (3)/scale industries. (4)/No error (5)

Ans (2): Use ‘Indian’ in place of ‘India’.


315. A recent RBI study shows (1)/that people prefer debit cards (2)/to credit cards (3)/while making payments.
(4)/No error (5)

Ans (5): No Error.


316. Our market research shows (1)/that there will be (2)/a huge demand by such (3)/products in the next few
years.(4)/No error (5)

Ans (3): Replace group of words ‘a huge demand by such’ by ‘huge demand for such’.
Look at the sentence: There is an increased, demand for organic produce these days.
317. The meeting cannot (1)/ be held since (2)/some executives are (3)/unlike to attend. (4)/No Error (5)

Ans (4):Here, an Adverb should be used to modify an Infinitive. Hence, ‘unlikely to attend’ will be the correct
substitution
318. Since the company has (1)/not took any action (2)/he had decided to (3)/file a case against it.(4)/No Error (5)

Ans (2): We use Past Participle form of a Verb (V3) in Present Tense. Hence, since the company has not taken … will
be form of Verb ‘take’.
319. The university has (1)/ not spent enough (2)/money to maintaining (3)/its valuable library. (4)/ No error (5)

Ans (3): In some sentences, an infinitive is used to express purpose or result.


Look at the sentences: He went to market to buy a cow. We came to realise our mistake. Hence, replace …. ‘money
to maintaining’ by ‘money to maintain’.
320. The survey found (1)/that most of the employees (2)/ has a positive view (3)/of the company. (4)/ No error (5)

Ans (3): The subject of the sub-ordinate clause of the sentence is in Plural Number that will take Plural Verb.
Hence, replace ‘has a positive view’ by ‘have a positive view’
321. The meeting will not end (1)/till the chairman (2)/gets approval from (3)/every members of the Board. (4)/No
error (5)

Ans (4): Singular Noun or Pronoun is used with each and every.
Hence, replace ‘every members of the board’ by ‘every member of the board’.
322. For millions of people (1)/his retirement from (2)/ cricket has been (3)/a greatest shock. (4)/No error(5)

Ans (4): Replace ‘a greatest shock’, because Article ‘the’ is used before Superfluous Degree. We can also use ‘a great
shock’.
323. The chairman has refused (1)/no to sanction (2)/ the required funds (3)/for the project. (4)/No error (5)

Ans (2): The chairman has refused to sanction … will be a correct sentences. The word ‘refuse has a negative
meaning. Use of ‘no’ before it is superfluous.
324. The interrogation made by (1)/ him hardly yield (2)/ any concrete conclusion (3)/ about the crime. (4)/ No
error (5)

Ans (2): The sense of the sentence shows past time. Hence, the interrogation made by him hardly yieded … will be
the correct usage.
325. What you had said (1)/ about the employees was (2)/ found to be correct (3) / but it could not be proved. (4)/
No error (5)

Ans (5): No Error.


326. When you had started (1)/ the work, you should (2)/ ensure that you (3)/ concentrate on it. (4)/ No error (5)

Ans (1): The sense of sentence is in Present Tense. Hence, when you start … will be the correct usage.
327. If you have good (1)/ control over breathing (2)/ you can float (3)/ on water effortlessly. (4)/ No error (5)

Ans (2): If you have good control over breath … will be the correct usage.
Look at the sentences:
His breath smelt of garlic.

Noun
We had to stop for breath before we got to top

Noun.
The way to the fort (1)/ was too difficult that (2)/ we could not reach (3)/ the farthest point. (4)/ No error (5)
Ans (2): The way to the fort was so difficult …. Is the correct usage. Such sentences express cause.
Structure:
Subject + Verb +too + adjective/ adverb + infinitive.
Look at the sentences:
He is too weak to run. (with ‘too’)
He is so weak that he cannot run. (without ‘too’)
328. When we heard the name (1)/ of our leader uttered (2)/ respectfully by the foreigners (3)/ we felt pride of him.
(4)/ No error (5)

Ans (4): We felt proud of him …. Is correct sentence.


329. It goes to his credit (1)/ that he was a clean man (2)/ in politics as well as (3)/ his person dealings. (4)/ No
error (5)

Ans (4): ‘…. His personal dealings’ is correct sentence, because personal (Adjective) should be used before dealings
(Noun).
330. His strictly discipline (1)/ has made him (2)/ very unpopular among (3)/all the employees. (4)/ No error (5)

Ans (1): His strict discipline … is correct sentence. The word strictly is an Adverb and it is not proper to use it
before a Noun.
331. Jamshedji knew that (1)/ an industrial revolution could (2) / only brought in the country (3) / by setting up
iron and steel industry. (4) / No error (5).

Ans (3): The sense of the sentence is in Passive Voice. Hence, … could only be brought in the country … will be
correct sentence.
332. He being the oldest son (1) / has requested us (2) / to look after the problem (3) / faced by his father. (4) / No
error (5)

Ans (2): The sentence shows past time. Hence, He being the oldest son requested us … will be correct usage.
333. In about five minutes, they would (1)/ cover the distance from the read to the point (2)/ where the waves
would (3)/ begin lick their feet. (4)/ No error (5)

Ans (2): Replace the group of words ‘cover the distance from the road to the point’ by ‘cover the distance by/along
the road to the point’.
Look at the sentences: He was walking along the road when he was attacked. It takes about five hours by road.
334. The devastating experiences (1)/ of many wars taught some countries (2)/ the necessary of pursuing peace (3)/
at the expense of nationalist egos. (4)/ No error (5)

Ans (3): Replace groups of words ‘the necessary (Adjective) of pursuing peace’ by ‘the necessity (Noun) of pursuing
peace’.
335. The former superstar recently (1)/ visit an orphanage, (2)/ fuelling speculation that (3)/ she was planning to
adopt a child. (4)/ No error (5)

Ans (2): The sentence shows a past event. Hence, it is proper to use Simple Past. Hence, replace group of words
‘visit an orphanage’ by ‘visited an orphanage’.
336. Accordance to a new study (1)/ taking steps to remain healthy and young (2)/ may help delay (3)/ the onset of
Alzheimer’s disease. (4)/ No error (5)

Ans (1): Here, ‘In accordance with / According to a new study’ should be used. It is a correct idiomatic expression.
337. A day after he was diagnosed with (1)/ swine flu, preventive measures were put in place (2)/ to ensure that
others were not (3)/ affected by the dreaded virus. (4)/ No error (5)

Ans (5): No error.


338. The administration has conclusive (1)/ that it is retailers who are (2)/ responsible for upsetting (3)/ the city’s
household budget. (4)/ No error (5)

Ans (1): The structure of a sentence in Present Perfect Tense is: Subject + have/has + V3 (Past Participle). Hence,
‘The administration has concluded’ should be used.
339. The assurances, unfortunately, (1)/ remained on paper, as (2)/ neither the Center or the state initiated steps
(3)/ for the development of the backward region. (4)/ No error (5)

Ans (3): Neither …. nor is a correct connective. Hence, ‘neither the Center nor the state initiate steps’ should be
used.
340. Sediment deposit along the coast (1)/ may be the primary reason for (2)/ the change in conditions, (3)/ but a
lot more remains to be understand. (4)/ No error (5)

Ans (4): Here, ‘but a lot more remains to be understood (V3) should be used.
341. A committee will be set up (1)/ to explore pros and cons by (2)/ a common fee structure and will (3)/ take a
final decision on it within a week. (4)/ No error (5)

Ans (2): Here, ‘to explore pros and cons of’ should be used.
342. The infection which causes (1)/ gums to bleed and teeth to fail out (2)/ results from the build up of (3)/ a
particular bacteria that is common to the most mouths. (4)/ No error (5)

Ans (5): No error.


343. My cousin’s wedding provided (1)/ me with the chance (2)/ to meet which relatives I (3)/ had not met for a
long time. (4)/ No error (5).

Ans (3): The group of words ‘to meet the relatives I’ or ‘to meet the relatives whom I’ should be used.
344. There are floods in this (1)/ region last year but (2)/ many of the victims have (3)/ not yet been compensated.
(4)/ No error (5).

Ans (1): The event shows past time. Hence, ‘There are floods in this’ by replace ‘There were floods in this’ should be
use.
345. We must ensure that (1)/ all our records (2)/ are computerised (3)/ for next year. (4)/ No error (5).

Ans (4): Preposition ‘the’ should follow the word ‘next’. Hence, replace ‘for next year’ by ‘for the next year/till the
next year should be used.
346. In order to impact (1)/ training to bank employees (2)/ we are setting up (3)/ centers at various location. (4)/
No error (5)

Ans (4): Replace ‘centers at various location’ by ‘centers at various locations’.


347. When Rahul reached (1)/ the office there was (2)/ nobody there so (3)/ he sat down to wait. (4)/ No error (5)

Ans (5): No error.


348. I shall have to check (1)/ my records to find out (2)/ how much cheques (3)/ have been collected so far. (4)/
No error (5)

Ans (3): Replace ‘how much (quantity) cheques’ by ‘how many (quantity) cheques’
349. Our first task as (1)/ board members is (2)/ to decide on what we (3)/ should spend these funds. (4)/ No error
(5)

Ans (5): No error.


350. She was lucky (1)/ to find a good paying (2) job as soon (3)/ as she graduated. (4)/ No error (5)

Ans (2): The group of words, ‘to find a well paid’ should be used.
351. For the last six months (1) Seema has been asking them (2)/ to refund the money but (3)/ there is been no
action. (4)/ No error (5)

Ans (4): Replace ‘there is been no action’ by ‘there is/has been no action’.
352. He promised to get (1)/ in touch with myself (2)/ as soon as he had (3)/ the information I needed (4)/ No error
(5)

Ans (2): Replace ‘in touch with myself’ by ‘in touch with me.
353. Mr. Gupta insisted (1)/ that he was (2)/ fully prepare (3) / to present the proposal. (4)/ No error (5).

Ans (4): Here, fully prepared (Adjective) should be used.


Look at the sentence: I was not prepared for all the problems it caused.
354. All this document (1)/ have been destroyed (2)/ by the fire (3)/ which occurred last month. (4)/ No error (5).

Ans (1): Here, Plural Subject i.e. All these documents …. Should be used.
355. On his next (1)/ official visit to Delhi (2)/ he plans to (3) / visit his old colleagues. (4)/ No error (5)

Ans (1): Here, During his next … should be used. The word during (Preposition) is used in the sense of – at some
point of time in a period of time.
Look at the sentence: I only saw him once during my stay in England.
356. Sunil refused to admit that (1)/ he did not know (2)/ what to operate (3)/ the new machine. (4)/No error (5)

Ans (3): Here, how to operate (in what way or manner) should be used.
Look at the sentence: He did not know how he ought to behave.
357. He has refused (1)/ not to take (2)/ the promotion although (3)/ he is eligible. (4)/ No error (5)

Ans (2): Here, to take … should br used. The meaning of the word refuse is: turn down; deny.
358. There has been (1)/ a fall from (2)/ the profits of (3)/ our company recently. (4) / No error (5)

Ans (2): Here, a fall in … should be used.


359. The decision to open (1)/ offices in shimla (2)/ were taken at (3)/ the last Board meeting./ (4) No error (5)

Ans (3): Here, the subject is singular. Hence, was taken at … should be used.
360. We were so late (1)/ than the meeting (2)/was almost over/(3)/ when we arrived. (4)/ No error (5)

Ans (2): Here, so … that should be used.


Look at the sentence: He is so weak that he cannot walk.
361. They waited patient (1)/ for the Chairman’s speech (2)/ to end in order (3)/ to begin the discussion. (4)/ No
error (5)

Ans (1): An Adverb is used to modify a Verb. Hence, They waited patiently … should be used.
362. Arjun was (1)/ not selected (2)/ for the post (3)/ inspite his hard work. (4)/ No error (5)

Ans (4): Here, inspite of his hard work should be used.


363. The majority of the population believes (1)/ that the stock market is recovered (2)/ and that it is the (3)/ right
time to buy shares. (4)/ No error (5).

Ans (2): When some work is completed in recent past, Present Perfect should be used. The structure of the sentence
is:
Subject + has /have + Past Participle (V3)
Hence, ’that the stock market has recovered’ or that the stock market is recovering’ will be correct usage.
364. Seema’s new office is close. (1)/ to her residence so (2) / she will not longer (3) /have to travel by train. (4)/ No
error (5)

Ans (3): Idiom ‘no /any longer’ is: used to say that something which was possible or true before, is not now. Hence,
‘she will no longer’ will be a correct usage.
365. To the present guidelines. (1) /the bank is required to obtain (2)/ a photograph from the person
(3) /who wishes to open an account. (4) /No error.

Ans (1): The group of words ‘To the present guidelines’ should be replaced by ‘Under the present
/latest/ new guidelines or According to the present/ latest /new guidelines.
366. Chetan puts in a lot (1)/of effort to ensure (2)/that the scheme was (3) /launched in March. (4)/No error.

Ans (3): Here, Present Indefinite (Passive) should be used. We notice a definite programme. Hence, ‘that the scheme
is launched in March’ will be a correct usage.
367. As our economy is (1)/experiencing a recession, banks (2) /have became very cautious (3) /about giving loans.
(4) /No error (5)

Ans (3): The structure of sentence in Present Perfect Tense is: Subject + has/have + V3
Hence, ‘banks have become very cautions’ will be a correct usage.
368. After retiring she spent (1)/most of her time teaching young (2) /doctors and motivating themselves (3) /to
work in rural areas. (4) /No error (5)

Ans (3): Here, Objective case should be used. Hence, ‘doctors and motivating them’ should be used.
369. Since this foreign bank has (1) /not performed very good this year (2) /it will not be setting (3)

/up any new branch. (4) No error (5)


Ans (2): Word ‘well’ is used as an Adverb. Hence, ‘not performed very well this year’ will be correct usage.
370. Our Chairman is deeply concerned (1) /about the environment (2) /and plans to take some steps to reduce the
(3) /pollution caused by our factories.(4)/No error (5)

Ans (4): The group of words ‘pollution caused by these/the factories’ will be a correct usage.
371. He was afraid that his parents (1)/would not allow him to (2) /make the film so he did (3)/not tell them about
it. (4) /No error (5)

Ans (5): No error.


372. I was previously posted (1) /in Singapore and it (2)/ took me a long time (3) /to accustom to the flood. (4)/ No
error (5)

Ans (4): Here, ‘to accustom myself to the food/to get accustomed to the food’ will be a correct usage.
373. Look! (1)/ somebody (2)/ had climbing up (3)/ that tree over there. (4)/ No Error (5)

Ans (3): Here, the work is in progress. Hence, Present Progressive i.e. is climbing up …should be used.
374. Sachin is one (1)/ of the greatest players (2)/ who has (3)/ played for India. (4)/ No Error (5)

Ans (3): Here, antecedent of who is players (Plural). Hence, who have (plural) … should be used.
375. I saw jack (1)/ at a party (2)/ a few month ago (3)/ and he did seems fine. (4)/ No Error (5)

Ans (4): To emphasise, we use do/ does/did.


Look at the sentences:
I do read English

V1
He did read English

V1
Hence, and he did seem fine should be used.
376. Betty has a (1)/ term paper to write (2)/ and she has a lab report (3)/ in finish too. (4)/ No Error (5)

Ans (4): Here, infinitive i.e. to finish too/ to cpmplete too … should be used.
377. If a hotel is (1)/ very expensive, it does not (2)/ mean that its (3)/ services is best. (4)/ No Error (5)

Ans (4): Here, service is fine or services are the best … should be used.
378. The knight adored his pride wife (1) /but he was in (2) /mortal fear of (3) /her fierce temper. (4)

/ No Error (5)
Ans (1): Here, The knight adored his proud wife … should be used. The word ‘pride’ is a Noun while; proud’ is an
Adjective that qualifies a Noun.
379. The book that (1) /the student want to borrow (2) /was not available (3) /in the library. (4) /No Error (5).

Ans (2): Here, Past Simple i.e. the student wanted to borrow … should be used.
380. The exam was (1) /conducted in the (2) /school library as there was (3) /minimise noise (4) /No Error (5).

Ans (4): Here, replace minimize (verb) noise by minimum (Adjective) noise.
381. My colleague to longed for (1) /the holiday to come (2) /so that she could be (3) /with her family again. (4)
/No Error (5)

Ans (1): The word long (Verb) means: to want something very much. Hence, My colleague too, was longing
for/longed for … should be used here.
382. I regrets (1) /to inform you that (2) /your application (3) /has been rejected. (4) /No Error (5)

Ans (1): Here, subject ‘I’ will agree with plural verb ‘regret’.
383. You can help (1)/ yourself to some biscuits, (2)/ although there are (3)/ a less left. (4)/ No Error(5)

Ans (4): The word less (determiner) is generally not used with countable nouns. Hence, fewer/ a few left should be
used here.
384. Each soldier were (1)/ praised for his bravery, (2)/ and was each (3)/ given a medal. (4)/ No Error (5)

Ans (1): Each soldier is a singular subject. Hence, Each soldier was … should be used.
385. The aim of (1)/ the organisation is to (2)/ educate the public about (3)/ the dangers of the smoking. (4)/ No
Error (5)

Ans (4): It is not proper to use article ‘the’ before smoking (uncountable). Hence, the dangers of smoking … should
be used here.
Look at the sentences: Would you like smoking? He is trying to give up smoking.
386. People who smoke (1)/ are not the only ones (2)/ to be affect (3)/ by lung cancer. (4)/ No Error (5)

Ans (3): Look at the passive of an infinitive: to consume => to be consumed. Hence, to be affected
…. Should be used here
387. There was no one (1)/ from America (2)/ between the many tourists (3)/ in the train. (4)/ No Error (5)

Ans (3): Here, among the many tourists …should be used. Connective ‘between’ is used for two persons or things.
388. Mahesh and Sachin live (1)/ next by the post office (2) / not far (3)/from the station. (4)/ No error (5)

Ans (2): Replace ‘next by’ by ‘next to’. It means following in order or importance after somebody/something.
389. After the tsunami (1) / they didn’t know (2)/ weather they (3)/ would be able to rebuild. (4)/ No error (5)

Ans (3): Replace ‘weather’ by ‘whether.


390. It’s been difficult (1)/to persuade (2)/ to the private sector (3)/ to invest in vaccines. (4)/ No error (5)

Ans (3): Remove ‘to’


Look at the sentence:
It will be difficult to persuade herself that life was not worth living.
391. Each of them (1)/along with their families and their pets (2)/is going to Goa (3)/for the summer vacation. (4)
/No error (5)

Ans (4): Replace ‘for the summer vacation’ by ‘during the summer vacation’.
392. He never attended lectures (1)/and used to borrow (2)/ the books of his friends (3)/ to study for the exam. (4)/
No error (5)

Ans (3): Replace ‘the books of his friends’ by ‘the books from his friends’. The word ‘borrow’ takes preposition
‘from’.
Look at the sentence: Members can borrow upto ten books from the library at any one time.
393. One of the best way (1)/ of improving (2)/ your English is to (3)/ read the language. (4)/ No error (5)

Ans (1): ‘One of’ id followed by a Plural Noun. Hence, One of the best ways … will be a correct sentence.
394. I was halting (1)/ at the station for (2)/ the train to arrive when (3)/ suddenly there was a loud noise. (4)/ No
error (5)

Ans (1): Replace group of words ‘I was halting’ by ‘I was waiting’.


395. I reached (1)/ lately for the meeting (2)/ as I got (3)/stuck in traffic. (4)/ No error (5)

Ans (2): The word ‘lately’ means: recently; in the recent past.
396. Unless you (1)/ work hard you (2)/ will not achieve (3)/ success in life. (4)/ No error (5)

Ans (5): No error.


397. The labourers working at (1)/ the construction site were (2) /very tiring and (3)/ wanted to rest. (4)/ No error
(5)

Ans (3): Replace group of words ‘very tring and’ by ‘very tired and’.
398. As it was (1)/ the boss’s birthday (2)/ all the employees wanted (3)/ to buy a gift to him. (4)/ No error (5)

Ans (4): Here, the word ‘gift’ should be followed by preposition ‘for’.
399. Priya got a (1)/ new job and she (2)/ happily told everyone (3)/ about them. (4)/ No error (5)

Ans (4): Here, the Objective Case for job will be ‘it’. Hence, replace ‘about them’ by ‘about it’
400. Being new to the country (1)/ myself decided to first take a (2)/ quick tour around (3)/ the place I was staying.
(4)/ No error (5)

Ans (2): Replace ‘myself’ by ‘I (Nominative case)’.


401. Since I had bought (1)/ quite a few items (2)/ the store keeper smiles (3)/ and gave me a discount. (4)/No
error (5)

Ans (3): Here, Simple Past should be used. Hence the storekeeper smiled should be used.
402. After waiting for (1)/ a very long time (2)/ the man finally (3)/ returned home frustrated. (4)/ No error (5)

Ans (5): No error.


403. Progress is impossible (1)/ without change, and those (2)/ who cannot change their minds (3)/ cannot change
nothing. (4)/ No error (5)

Ans (4): Replace group of words ‘cannot change nothing’ by ‘cannot change anything’. The sense of the sentence
suggests so.
404. She was allergic (1)/ to some medicines (2)/ and informed (3)/ the doctor about it. (4)/ No error (5)

Ans (5): No error.


405. He walked as faster (1)/ as he could so that (2)/ he would not (3)/ miss the train to work. (4)/ No error (5)

Ans (1): ‘As fast as’ will be correct usage.


406. She shared (1)/ all her secrets with (2)/ Suman as they had been (3)/ friends with childhood. (4)/ No error (5)

Ans (4): Replace group of words ‘friends with childhood’ by ‘friends since childhood’. The preposition ‘since’ is
used to show from a time in the past until a later past time or until now.
407. Gautam did not care (1)/ so many about (2)/ anything else as much (3)/ as he cared for his dog. (4)/ No error
(5)

Ans (2): Replace ‘so many about’ by ‘so much about’.


408. You cannot change (1)/ people, but you (2)/ can definitely (3)/ change own. (4)/ No error (5)

Ans (4): Replace ‘change own’ by ‘change yourself’ (the reflexive form of you).
409. Veena wanted to (1)/ become a surgeon (2)/ and worked very hardly (3)/ to achieve this. (4)/ No error (5)

Ans (3): The word ‘hardly’ means: almost none. Hence, replace ‘and worked very hardly’ by ‘and worked very hard
(Adverb)’.
410. Krishna ran to the (1)/ nearing grocery store to (2)/ buy biscuits as his parents (3)/ were expecting guests. (4)/
No error (5)

Ans (2): Replace ‘nearing grocery store to’ by ‘near-by grocery store to’.
411. As soon so (1)/ he came home, (2)/ he showered and got (3)/ ready to go out again. (4)/ No error (5)

Ans (1): ‘as soon as’ is the correct form of correlative.


412. She failed to remember (1)/ that it was Rema’s birthday (2)/ and did not buy (3)/ a present for her. (4)/ No
error (5)

Ans (5): No error.


413. The employee was (1)/ asked to leave her job (2)/ as she was not (3)/ performing well. (4)/ No error (5)

Ans (3): Here, Past Perfect Continous i.e. as she had not been … should be used.
414. In school (1) / we have to enrolled (2)/ in sports (3)/ and music classes. (4)/ No error (5)

Ans (2): ‘The form of infinitive is: to + V1. Hence, we have to enroll … should be used.
415. This report (1)/ highlight on (2)/ the need for (3)/ further research. (4)/ No error (5)

Ans (2): Here, use of preposition ‘on’ is superfluous.


416. My best friend (1)/ left the country (2)/ I hope to hearing (3)/ from her again soon. (4)/ No error (5)

Ans (2): Here, Present Perfect i.e. has left the country … should be used.
417. She is pretending (1)/ to be sick (2)/ because she don’t (3)/ want to study. (4)/ No error (5)

Ans (3): In Simple Present (Negative) ‘doesn’t’ is used with third person Singular Number.
Hence, because she doesn’t … should be used.
418. While I am (1)/ doing the house works, (2)/ I like to listen to (3)/ music on the ratio. (4)/ No error (5)

Ans (2): Here, doing the house work should be used.


Look at the sentences: I have plenty of work to do. (✓), I have plenty of works to do. (X)
419. I was very lucky (1)/ that day (2)/ and catched (3)/ a lot of fish. (4)/ No error (5)

Ans (3): The past form of catch is caught.


420. Several banks (1)/ are shown interest (2)/ in the proposed development (3)/ of the public sector. (4)/ No error
(5)

Ans (2): Here, Present Continuous or are showing interest … should be used.
Look at the structure: Subject + am/are/is + V- ing.
421. In order (1)/ to be successful (2)/ you must worked (3)/ very hard. (4)/ No error (5)

Ans (3): Modals must, should, would are followed by infinitive (without to).
Look at the sentences: I should help you. You must respect the leader.
Hence, you must work … should be used.
422. As a child (1)/ I always want to (2)/ have a bicycle (3)/ of my own. (4)/ No error (5)

Ans (2): Here, I always wanted to (Simple Past) … should be used.


423. The new assignment is (1) / more challenging that (2) / much of the (3)/ earlier assignments. (4)

/ No error (5).
Ans (3): Many will be used in place of much. Much is used with Singular Noun; many is used with Plurals.
For example,
i. I do not know many of your friends.


Plural
ii. You can have as much of the milk as you like.


Singular
In the second sentence, milk is uncountable material Noun. Soit is always used in Singular Number
In the given sentence assignments is in Plural Number. Therefore, many should be used instead of much.
424. When the doctors found (1) / that the player has taken (2) / prohibited medicines, he reported (3)

/ the mater to the team manager. (4) / No error (5)


Ans (2): Instead of has, had must be used in part (2) of the sentence. From the use of found in the part (1), it
becomes clear that the sentence is in Past Tense. Again, Taking of prohibited medicine had been taken place before
doctors found it out. Therefore, that action (taking of prohibited medicines) should be in past Perfect Tense and in
no way in Present Perfect Tense.
425. We are sure that (1) / they will definitely help us (2) / if we approach (3) / them well in advance.
(4) / No error (5).

Ans (2): Use of definitely after the use of sure is superfluous as both of these words bear the same meaning.
426. They now claim that (1) / they would have (2) / guided us if we (3) / would have requested them. (4) / No
error (5)

Ans (4): In part (4) of the sentence had must be used in place of would have. This type (given) of sentence follows
the structure given below.
SUBJECT + WOULD + HAVE + V3 IF + WE + HAD + V3
That is, in the subordinate clause (clause starts with if), had is used and not would have.
427. None of the passengers were (1) / aware of what (2) / was going to happen (3) / in the next two hours. (4) /
No error (5)

Ans (1): Was should be used in place of were. None is always followed by a Plural Subject but Singular Verb. Here
also the Verb must be in Singular number. For example,
None of the boys was happy (not were happy).
428. Despite of their best efforts (1)/ they failed to retain (2)/ the contract due to (3)/ unwanted political
interference. (4)/ No error (5)

Ans (1): The word despite takes no preposition. Its synonym is ‘inspite of’.
Look at the sentence:
Despite applying for hundreds of jobs, he is still out of work. Hence, despite of their best efforts should be
replaced by despite their best efforts.
429. We had decided (1)/ to scrap the project (2)/ but the chairman insisted with (3)/ its continuation on the same
terms. (4)/ No error (5)

Ans (3): The word ‘insist will take Preposition ‘on’.


Look at the sentence:
They insisted on a refund of the full amount. Hence, ------ but the chairman insisted on its continuation … will be
correct sentence.
430. Because of his prolonged illness (1)/ he could not concentrate (2)/ on his studies although (3) he was very
much desired to so. (4)/ No error (5)

Ans (4): Here, Passive form of the sentence is improper. Hence, he was very much desired to do so should be
replaced by he desired very much to do so.
431. You must realize (1)/ how importance it is (2)/ to give away to the needy (3)/ whatever you possess in excess.
(4)/ No error (5)

Ans (2): The use of word importance (Noun) is incorrect. It should be replaced by important (Adjective).
432. Most of the travelers experienced (1)/a shock when they arrived (2)/ at the museum only to find (3)/ that it
had been burgled. (4)/ No error (5)

Ans (3): The Adverb only should be positioned after to find.


433. How people do (1)/ to earn their bread (2)/ is dependent largely on (3)/ the locality where they live. (4)/ No
error (5)

Ans (1): In the sentence, how people do should be replaced by what people do.
434. He took a sharp knife (1)/ and torn the covering (2)/ made of cardboard (3)/ but could not open the packet.
(4)/ No error (5)

Ans (2): The principal clause is in Simple Past and connective and has been used. Hence, the succeeding part will
also be in Simple Past. Hence, he took a sharp knife and tore the covering … will be correct sentence.
435. The security force posted (1)/ at the entrance is so strict (2)/ that even the top boss (3)/ cannot be entered
without frisking. (4)/ No error (5)

Ans (4): …. But even the top boss cannot enter without being frisked will be correct usage.
436. The moment he tried (1) to escape from the (2) clutches of the policemen (3)/ they knocked him down. (4)/ No
error (5)

Ans (5): No error.


437. Some people have (1)/ generously contributed to the welfare fund (2)/ but they wanted that there names (3)/
should not be published. (4)/ No error (5)

Ans (3): There should be replaced by there (Possessive case).


438. We were aware that (1)/ she had received many awards (2)/ and was felicitated on (3)/ account for her
wisdom. (4)/ No error (5)

Ans (4): The correct form of idiom is –on account of.


439. They are aware of (1)/ the risk involved in (2)/ investing the entire funds (3)/ in only one scheme. (4)/ No
error (5)

Ans (4): They are aware of … will be correct sentence.


440. When we were reached (1)/ at the gate of the park (2)/ it was too dark (3)/ to see the new entrants. (4)/ No
error (5)

Ans (1): Use of Passive Voice is incorrect. Hence, when we reached … will be correct sentence.
441. The soldiers were supposed (1)/ to sought shelter inside (2)/ the nearby factory (3)/ to save them selves. (4)/
No error (5)

Ans (2): Infinitive takes V1 form. Hence, the soldiers were supposed to seek shelter … will be correct sentence.
442. Being a good swimmer (1)/ he could keep (2)/ himself floating (3)/ the surface of the lake (4)/ No error (5)

Ans (3): The word float will take Preposition on.


443. They are very senior (1)/to me; so I (2)/dare not play a joke (3)/on them. (4)/ No error (5).

Ans (1): Very is used before a positive degree Adjective; where as much is used before a comparative degree
Adjective.
For example:
i. He is very good

↓↓
Very Adj
(Adv.) (Positive Degree)
ii. He is much better than you.

↓↓
MUCH ADJ
(Adv.) (Comparative Degree)
In the given sentence senior is a comparative degree Adjective. So much will be uses in place of very.
444. He asked me (1)/that why I was (2)/not preparing for the (3)/civil services exam.(4)/No error (5).

Ans (2): We do not use any Conjunction before wh-question in indirect Narration.
For example,
Correct – He asked me why I was late.
Incorrect – He asked me that why I was late. Therefore, in the given sentence we must to omit that.
445. The poet describes about (1)/the beauty of a landscape (2)/in a very (3)/attractive way. (4)/No error (5).

Ans (1): We use


DESCRIBE/DISCUSS/EXPLAIN + SOMETHING.
For example,
He explained this poem
↓↓
EXPLAIN SOMETHING (V2)
But,
i. Describe something to somebody
ii. Explain something to somebody
iii. Discuss something to somebody. He described the event to me

↓↓↓↓
Describe some To somebody (V2) -thing
Therefore, in the given sentence, about should not be used.
446. Have you (1)/turned detective (2)/that you keep you eye (3)/on me like this? (4)/No error (5).

Ans (5): No error.


447. She asked him (1)/what is was that made him (2)/so much stronger and braver (3)/than any man. (4)/No
error (5).

Ans (4): When compare between two things or being belong to the group, we use any other. But, when the things or
beings belong to different groups, we use any only.
For example,
i. Ganga is more important that any other river in India.
ii. Ganga is more important that any river in Bangaladesh.

In the first (i) case, the comparison is between Ganga and any other river in India. Here, both, the rivers belong to
the same group ‘Indian rivers’; whereas, in the second (ii) case, comparison is between Ganga (Indian river) and
any river inBangaladesh. So in the second case, only any other should be used in place of any.
448. The more enlightened (1)/ factory owners (2)/ build homes for their (3)/ workers in Britain in the 19th century.
(4)/ No Error (5).

Ans (3): Event shows past time. Hence, Simple Past i.e. built homes for their … should be used.
449. Most criminals (1)/ realises that they may be caught, (2)/ but nevertheless (3)/ they are prepared to take the
risk. (4)/ No Error (5).

Ans (2): Here, the subject ‘Most criminals’ is Plural. Hence, Plural verb i.e. realize that … should be used.
450. Strictly speaking, (1)/ spiders are not insects, (2)/ although most people (3)/ think them are. (4)/ No Error (5).

Ans (4): Here, Nominative case i.e. think they are … should be used.
451. The actress is also (1)/ widely respected in (2)/ the music world (3)/ and deserving so. (4)/ No error (5).

Ans (4): Here, and she deserves so (Simple Present) should be used.
452. The story was (1)/ inspired by (2)/ a chance meeting (3)/ with an old Russian duck. (4)/ No Error (5).

Ans (3): Here, a meeting by chance/ an accidental meeting …. Should be used.


453. The travel agency made (1) / all the arrangements (2) / for our journey (3) / for England. (4) / No error (5)

Ans (4): In the sentence ‘for England’ should be replaced by ‘to England’. He went to England for higher studies.
454. The boys made up (1) / when the (2) / owner of the (3) / garden appeared. (4) / No error (5) Ans (1): If two
actions happened in the past, it may be necessary to show which action happened earlier that the other. The Past
Perfect is mainly used in such situation. The Simple Past is used in one action that has happened later and Past
Perfect is used in the other

For example,
When I reached the station the train had started. I had written the letter before the arrived.
Hence, the boys had made up --- will be the correct usage.
455. Gandhiji was a (1) / man who (2) / become a legend (3) /in his own time. (4) / No error (5)

Ans (3): The principal clause of the sentence, ‘Gandhiji was a man’ is in Past Tense.
Hence, who became ---will be the correct usage.
456. The central government has (1) / refused to meet (2) / all the demands (3) / off its employees in toto. (4) / No
error (5)

Ans (5): No error.


457. The questions was (1) / so difficult that (2) / nobody were (3) / able to answer it. (4)/No error (5)

Ans (3): ‘No body’ has a singular sense and it takes a singular verb.
458. The committee has made (1)/ significant changes in (2)/ the rules which (3)/ will help to banks. (4)/ No error
(5)

Ans (4): It is improper to use Preposition ‘to’ here. Hence, replace ‘will help to banks’ by ‘will help the banks’.
459. This project which is funded (1)/ by the United Nations (2)/ has helped over four (3)/ lakh Indians overcome
poverty. (4)/ No error (5)

Ans (5): No error.


460. The merger talks have (1)/ taken longer that (2)/ we anticipated because (3)/of the financial crisis. (4)/ No
error (5)

Ans (2): We use ‘than’ instead of ‘that’ in Comparative Degree. Hence, replace ‘taken longer that’ by ‘taken longer
than’.
461. His share of (1)/the money from the sale (2)/off the property will (3)/ be given to charity. (4)/No error (5)

Ans (3): Replace group of words ‘off the property will’ by ‘of the property will’. Off shows distance or separation.
Look at the sentences:
I fell off the ladder.
There is a bathroom off the main bedroom.
462. The number of tourists (1)/ is expected to rise (2)/ from four to ten percent (3)/ in the next ten years. (4)/No
error (5)

Ans (5): No error.


463. When he asked (1)/ why the machines (2)/ were idle we told (3)/ him it was broken. (4)/No error (5)

Ans (4): The word ‘machines’ is in Plural Number. Hence, its Pronoun will able in Plural Number
i.e. they. Hence, replace ‘him they were broken’.
464. We informed customers (1)/ that there would be (2)/ no interrupting in (3)/ our bank services tomorrow. (4)/
No error (5)

Ans (3): The word ‘interruption’ is a Noun and the word ‘interrupting’ is an Adjective. As the sense of the sentence
suggests, it will br proper to use Noun here. Hence, replace ‘no interrupting in’ by ‘no interruption in’.
465. To train its newly (1)/ recruit staff the company (2)/ has developed an (3)/ online training programme. (4)/No
error (5)

Ans (2): Replace group of words ‘recruit staff the company’ by ‘recruited staff the company’. The word ‘newly’ is
wrongly followed by ‘recruit’ that is Noun. An Adverb qualifies an Adjective.
466. A person whose wishes to (1)/ avail of this facility (2)/ should not have been (3)/ a loan defaulter in the past.
(4)/No error (5)

Ans (1): The word ‘who’ (Pronoun) is used to show which person or people you mean.
Look at the sentence:
The people who called yesterday want to buy the house. Hence, replace ‘A person whose wishes to’ by ‘A person
who wishes to’.
467. Graduates have experience (1)/ in the IT industry (2)/ are in great demand (3)/ in the finance sector. (4)/ No
error (5)

Ans (1): Replace group of words ‘Graduates have experience’ by ‘Graduates having experience’ or ‘Graduates with
experience’.
468. Removing seeds from (1)/cotton plants was (2) / a slowest job until (3) /Eli Whitney invented the cotton gin.
(4) / No Error (5)

Ans (3): Article ‘the’ is used before Superlative degree. Hence, a slow job until/ the slowest job until … should be
used here.
469. India is a land of (1)/great political leaders (2) / who ruled the country effectively (3) /and also by protecting
its national interest.(4) / No Error (5)

Ans (4): Here, Past Simple i.e. and also protect its national interest … should be used.
470. The climbers will try again (1)/to reach the summit of the mountain (2) / their chances of doing so are (3)
/more better than they were last week. (4) / No Error (5)

Ans (4): Here, there is an error of double comparatives. Hence, much better than …/better than
… should be used.
471. Most birds sings (1)/only in daylight (2) / one notable exception (3) /being the nightingale. (4) / No Error (5)

Ans (1): Here, Most of the birds (Plural) sing… should be used.
472. In the eve of the election (1) / there was heavy (2) / political security (3) / in the state. (4) / No Error (5)

Ans (1): Here, On the eve of the election … should be used.


473. The teacher thought (1) / the children would (2) / be happy for (3) / clearing away their toys. (4)

/ No Error (5)
Ans (4): Here, clearing up their toys … should be used. Here, cleaning up their toys can also be used.
474. Neither Alice or Mary remembers (1) / what happened before (2) / the car crashed (3) / into the wall / No
Error (5)

Ans (1): Neither … nor is correct form of correlative.


475. This is the women (1) / that always wears (2) / a black shawl (3) / to work. (4) / No Error (5)

Ans (4): Here, at work/while working should be used


476. Many leading members (1) / of the opposition party (2) / has tried to justify (3) / their decisions.

(4) / No Error (5)


Ans (3):. Here, subject (Many leading members) is Plural. Hence, have tried to justify … should be used.
477. The view of (1) / the manufacturing and (2) / tourist industries is that (3) / the economy is improving. (4) / No
Error (5)

Ans (5): No error.


478. I did not (1) / went there (2) / to buy book or remnants (3) / for you. (4) / No error (5)

Ans (2): If an Auxiliary (like do, did, does, can, could, may, might, ought, dare not, need not, etc.) is followed by a
verb that verb must be in the form.
For example,
i. He did not do it.
↓↓
Auxiliary V1
In the given sentence also, went should be replaced with go (V1).
479. After listening to (1) / his advice, I (2) / decided not to (3) / go abroad for research. (4) / No error (5)

Ans (5): No error.


480. No other (1) / typist in this (2) / office is as (3) / slow as her. (4) / No error (5)

Ans (5): No error.


481. From thirty and (1) / forty percent of the (2) / people of this area (3) / suffer from jaundice (4) / No error (5)

Ans (1): If two words are preceded by from, these words are connected by to.
For example,
i. From Delhi to Mumbai
ii. From four to ten

In this way, we should use to in the place of and.


482. The director (1) / asked from me (2) / what I was (3) / doing in the cabin. (4) / No error (5)

Ans (2): Ask is a transitive Verb ans so, it is directly followed by an object.
For example,
He asked me my name.
↓↓
ASKED OBJ
As per this rule we should not use from in between asked and me.
483. Some people feels (1) / that with local language (2) / no progress (3) / is possible. (4) / No error (5)

Ans (1): Some people is a Plural Subject. Therefore, Verb used for it also must be in Plural Number. So, feels
should be replaced with feel.
484. Dandekar could not (1) / decide as to which (2) / article he should (3) / buy from the shop. (4) / No error (5)

Ans (2): We need not use as to. Use of as to distrots the meaning. So, in place of decide as to which we should use
decide which.
485. Having failed to (1) / got good results (2) / in her first attempt (3) / she did not try again. (4) / No error (5)

Ans (2): If a Verb is used after to, that Verb must be in the first form. To-infinitive takes the structure TO +V1. So,
got, in the given sentence should be replaced with get (V1).
486. She does not understand (1) / that how one can be employed (2) / as stenographer (3) / without the necessary
qualification. (4) / No error (5)

Ans (2): We cannot use that and how / where / when/ what together.
For example,
Incorrect- He asked me that how I had done that.
Correct-He asked me how I had done that.
Therefore that should not be used in the given sentence.
487. A few word of (1) / gratitude are enough (2) / to express your (3) / feeling sincerely. (4) / No error (5)

Ans (1): A few is used with a Countable Noun and that Countable Noun must be in Plural Number. Therefore,
instead of a few word, we should be use a few words.
488. The roads are flooded (1) / because it has raining (2) / contentiously for the (3) / last two days.

(4) / No error (5)


Ans (2): The use of for the last two days signifies that the sentence is in Present Perfect Continuous Tense. The
structure of Present Perfect Continuous Tense is : Subject + has /have
+been + V4.
So, has raining should be replaced with has been raining.
489. Sunita closed the (1) / window grill provided that (2) / her child might (3) / not fall down. (4) / No error (5)

Ans (2): “Window grill provided that” should be replaced with window grill provided. This should be done to
make the sentence meaningful.
490. He has never (1) / allowed and will (2) / never allow such practices (3) / in this office. (4) / No error (5)

Ans (3): Never should be replaced with not. Never should be repeat sentence which contains another negative
expression.
491. It was very kind (1) / about him to (2) / help us financially (3) / inspite of his poor economy .(4)

/ No error (5)
Ans (2): About him should be used in the place of about him to.
492. A large number of (1) / her class mates called for (2) / the hospital when (3) / she was there. (4) / No error (5)

Ans (2): Called for means “to demand or require”. Called on means “to visit”. In the given sentence Called on
should be used in place of called for.
493. When I called him yesterday, (1)/ he offered to donate (2)/ a handsome sum to (3)/ the flood relief fund. (4)/
No error (5)

Ans (1): Idiom call on means: to visit; to formally invite somebody or ask to speak etc. Hence, ‘when I called on him
yesterday’ will be the correct answer usage.
494. The student which (1)/ you had thought (2)/ so highly of has (3)/ failed to pass the examination. (4)/ No error
(5)

Ans (1): ‘Whom’ is used instead of ‘who’ as an object of a Verb or Preposition.


Look at the sentence:
The author whom you criticized in your review has written a reply. Hence, The student whom should be used.
495. Government took strict action (1)/ against the doctors on strike (2)/ but they refused to (3)/ resume to work.
(4)/ No error (5)

Ans (4): Here, use of Preposition ‘to’ is superfluous. Hence, resume work/duty should be used.
496. If a person has been given (1)/ diplomatic immunity than he (2)/ cannot be arrested on a (3)/ foreign land
under any circumstance. (4)/ No error (5)

Ans (4): ‘Any’ is used as determiner with uncountable or plural noun in Negative sentences. Hence, foreign land
under any circumstances (Plural) should be used.
497. A group of birds (1)/ migrate from southern part (2)/ of the country to the (3)/ Northern part during summer.
(4)/ No error (5)

Ans (2): Article ‘the’ should be used. Hence, migration from the southern part should be used.
498. The constable said that (1)/ the prisoner seize a (2)/ fully loaded gun from a policeman (3)/ and shot the
prosecutor. (4)/ No error (5)

Ans (2): Here, the Reporting Verb is in Past Tense therefore Reported Speech should also in Past Tense. Hence, the
prisoner seized/snatched a should be used.
499. It will not be possible for you (1)/ to catch the train on time (2)/ because the nearest railway station (3)/ is at
ten kilometers away. (4)/ No error (5)

Ans (4): To show distance, away is used in the following manner: The beach is a mile away. Hence, is ten
kilometers away should be used.
500. Each of the survivors of the Tsunami (1)/ have been offered free (2)/ psychological consultation to ease their
trauma, (3)/ by some of the top consultants. (4)/ No error (5)

Ans (2): After Each of Plural Noun or Pronoun is used. It takes, a Singular Verb. Hence, has been offered free
should be used.
501. Considering about her good credentials, (1)/ the manager offered her a job (2)/ in his organisation (3)/ despite
the lack of experience. (4)/ No error (5)

Ans (1): Considering her good credentials should be used. Use of Preposition ‘about’ is Superfluous.
502. Although he has been (1)/ winning the elections (2)/ over the years, this year his popularity (3)/ has
substantially reduced. (4)/ No error (5)

Ans (5): No error.


503. A man was sleeping (1)/ at night in his cabin (2)/ that suddenly (3)/ his room filled with light. (4)/ No error
(5)

Ans (3): Here, when suddenly … should be used.


504. For hours the secretary ignoring them, (1)/ hoping that the couple would (2)/ finally be discouraged (3)/ and
go away. (4)/ No error (5)

Ans (1): Here, The secretary ignored them for hours … should be used.
505. Working together for the first time, (1)/ the father and son developed concepts of how (2)/ they could
accomplish the task and (3)/ how the obstacles could be overcome. (4)/ No error (5)

Ans (4): Here, father and son developed … should be used. When nouns are used in pairs, no article is used.
506. Once upon a time there was (1)/ a water-bearer who had two large pots, (2)/ each hung on each end of a pole
(3)/ who he carried across his neck. (4)/ No error (5)

Ans (4): Here, which he carried across his neck … should be used.
507. Often when we face obstacles (1)/ in our day-to-day life, our hurdles (2)/ seemed very small in comparison
(3)/ to what many others have faced. (4)/ No error (5)

Ans (3): The sentence shoes fact and present time. Hence, seem very small in comparison (Present Simple) ….
Should be used.
508. The man ran (1)/ to a tree, climbed (2)/ on to it and (3)/ sit on a branch. (4)/ No error (5)

Ans (4): The event shows past time. Hence, Past Simple i.e. sat on a branch should be used here.
509. A young man was standing (1)/ on the road proclaiming (2)/ that he had the more (3)/ beautiful car in the
whole world. (4)/ No error (5)

Ans (3): Here, Superlative Degree i.e. that he had the most … should be used.
510. The girl sat down (1)/ for dinner but refused to (2)/ eat until (3)/ her father came home. (4)/ No error (5)

Ans (5): No error.


511. The only thing that (1)/ the couple worried (2)/ on was, whether their (3)/ happiness would last forever. (4)/
No error (5)

Ans (3): Here, about was … should be used.


512. One day, the King woke up (1)/ earlier than usually (2)/ to take a stroll (3)/ around his palace. (4)/ No error
(5)

Ans (2): Here, earlier than usual (Adjective) should be used.


513. I have criticize (1)/ the remarkable book (2)/ because I benefited (3)/ from reading it. (4)/ No error. (5)

Ans (1): The error lies in part (1) of the sentence. ‘I have criticize’ should be replaced with ‘I did not criticize’. It will
make the sentence meaningful.
514. As Arundhati Roy (1)/ in her foreword write (2)/ John offers (3)/ untold stories of people. (4)/ No error. (5)

Ans (2): The word ‘write’ is in Present Indefinite Tense. It should be replaced with ‘has written’ as Present Perfect is
used to express post action whose time is not given and not definite.
515. Responsibilities includes (1)/ working with the editors (2)/ on all aspects (3)/ of the editorial process. (4)/ No
error. (5)

Ans (1): The word responsibilities is a Plural subject. It will take Plural Verb.
516. We build a simple model (1)/ to test whether (2)/ there is a phase change (3) in the Indian economy. (4)/ No
error. (5)

Ans (5): No error.


517. Our results (1)/ indicate that (2)/ 1991 marked (3)/ the end of an era. (4)/ No error. (5)

Ans (5): No error.


518. While grouping the words (1)/ care have been taken (2)/ to include the role (3)/ grammar plays in a language.
(4)/ No error. (5)

Ans (2): The word care is singular and it will take singular verb. Hence, ‘care have been taken’ should be replaced
with ‘care has been taken’.
519. Between the years (1)/ 1952 to 1962 (2)/ I worked in the field (3)/ as sub-divisional officer. (4)/ No error. (5)

Ans (5): No error.


520. The Chief Minister (1)/ was obliged (2)/ to honour the promise (3)/ made during the contest. (4)/ No error. (5)

Ans (5): No error.


521. Distrust seems (1)/ to be a factor (2) borne out of (3)/ prevailing circumstances. (4)/ No error. (5) Ans (3): The
phrase ‘borne out of’, should be replaced with ‘born of ‘which means coming from the specified type of parents,
background, origin etc. For example.

Her socialist beliefs were born of a hatred of injustice.


522. As the minister was (1)/ very intelligent and hardworking (2)/ the King appointed him (3)/ as his chief
advisor. (4)/ No error (5)

Ans (5): No error.


523. Humour is a better way (1)/ to provide social commentary on (2)/ controversy issues and some movies (3)/ do
that successfully. (4)/ No error (5)

Ans (3): Here, Adjective form of controversy should be used because issues is a Noun. Hence, controversial issues
and some movies … is a correct usage.
524. The comedian enthralled (1)/ everybody with his quick witty as (2)/ he had chosen just the right topic (3)/ to
warm up to the crowd. (4)/ No error (5)

Ans (2): The word witty is an Adjective while wit (Noun) should be used.
525. The superstar reveal that (1)/ he had been turning (2)/ the controversial security measures into (3)/ a public
relations opportunity. (4)/ No error (5)

Ans (1): Here, The super star revealed that or Simple Past should be used.
526. An old tiger living in (1)/ the forest was not strong enough (2)/ to hunt animals and starved (3)/ for much
days. (4)/ No error (5)

Ans (3): Here, Infinitive form of verb i.e. starve should be used.
527. My singing style was unique, (1)/ but it would have been nothing (2)/ without the contribution of (3)/ the
brilliant music directors. (4)/ No error (5)

Ans (4): It is improper to use the.


528. The boy was almost (1)/ asleep when a ball (2)/ flies across the garden (3)/ and hit him on the foot. (4)/ No
error (5)

Ans (4): The event shows past time. Hence, Flew across the garden will be a correct usage.
529. At last Amar found (1)/ Naresh under the shade (2)/ of a tree and brought him (3)/ back on the city. (4)/ No
error (5)

Ans (4): Here, back in the city should be used.


530. Mother sat through her chair (1)/ by the fire to read her book (2)/ but her ten little boys were (3)/ too noisy.
(4)/ No error (5)

Ans (1): Here, Mother sat in /on her chair or Mother was sitting in/on her chair … should be used.
531. Since the lion was (1)/ very illness, all the other animals (2)/ in the forest gave the lion (3)/ a lot of medicines.
(4)/ No error (5)

Ans (2): Here, Very ill, all the other animals should be used. The word ill is an Adjective while
illness is a Noun.
532. People who intend (1) / to visit the tourist spots (2) / are always thrilling (3) / to see the scenario here. (4) / No
error (5)

Ans (3): Use Passive Voice. The use of Present Continuous is improper.
Look at the sentence:
He was thrilled at the prospect of seeing them again. Hence, ‘…. Are always thrilling’ should be replaced by ‘…. are
always thrilled’.
533. In such delicate matters, (1) / we often go with (2) / his advice as the has (3) / been handling such cases
effectively. (4) / No error (5)

Ans (5): No error.


534. You should think (1) / of all the possibilities (2) / before you take (3) / any decision. (4) / No error (5)

Ans (2): Use Preposition ‘about’ in place of Preposition ‘of’.


Look at the sentences:
What did you think about the idea?
535. He was too tired that (1) / he could not cross (2) / the street even with (3) / the help of a porter.

(4) / No error (5)


Ans (1): Use ‘he was so tired …’ in place of ‘he was too tired…’.
Look at the sentences:
He was too tired to walk. (with ‘too’)
He was so tired that he was not able to walk. (without ‘too’)
536. Your desire to (1) / meet the President (2) / without prior appointment (3) / cannot be fulfill.

(4) / No error (5)


Ans (4): Use ‘cannot be fulfilled’ (Passive Voice) in place of ‘cannot be fulfill’.
537. Whenever man attains fame, (1) / his person qualities are (2) / imitated by others who (3) / are close to him.
(4) / No error (5)

Ans (2):‘… his person qualities’ should be used in place of ‘… has person qualities’. The Adjective form of person is
personal and ‘qualities’ is a Noun.
538. Rivers, mountains and deep forests (1) / are the places (2) / mostly like by (3) / people living in urban areas.
(4) / No error (5)

Ans (3): Use ‘mostly liked by …’ in place of ‘mostly like by’.


539. When we visited his office (1) / we found that (2)/ he was sipping coffee (3) / with some of his colleagues. (4)
/ No error (5)

Ans (5): No error.


540. For giving up (1) / the bad habit of smoking (2) / use of chewing gum or (3) / similar other method can be
helped. (4) / No error (5)

Ans (4): Use Active Voice. Hence ‘… can help’ will be a correct usage
541. His obviously reluctance (1) / was viewed seriously by (2) / his superiors and (3) / he was suspended. (4) /
No error (5)

Ans (1): The word ‘obviously’ is an Adverb. The word ‘reluctance’ is Noun which should be preceded by an
Adjective. Hence, ’obvious reluctance’ is correct usage.
542. As they watched (1) / the football match (2) / the huge crowd (3) / chant in unison. (4) /No error (5)

Ans (4): Here, Past Simple Tense should be used. Hence, chanted in unison – will be correct usage as both events
show past time.
543. The noise was (1) / so faintly that (2) / one had to strain (3) / one’s ears to hear it. (4) / No error (5)

Ans (2): Here, Adjective i.e. so faint that … should be used.


544. When he found out that (1) / the girl had escaped (2) / he was absolute (3) / irritated and furious. (4) / No
error (5)

Ans (3): An Adverb modifies a Verb and also an Adjective. Hence, he was absolutely (Adverb) … will be a correct
usage.
545. The weather is (1) / much more warmer (2) / than it was (3) / a few days ago. (4) / No error (5)

Ans (2): There is an error of double comparatives. Hence, much warmer – should be used.
546. A vast numbers (1) / of people greeted (2) / the film star on his arrival (3) / at the airport. (4) / No error (5)

Ans (1): Here, ‘A vast number of’ should be used.


547. Time the concert ended, (1) / the crowd clapped (2) / and cheered (3) / enthusiastically. (4) / No error (5)

Ans (1): Here, when the concert ended…should be used.


548. The students blamed (1) / their professor for (2) / their late arrival (3) / in the concert. (4) / No error (5)

Ans (3): Here, professor is in singular number. Hence, his late arrival … should be used.
549. We have many rooms (1) / in our house, (2) / several of which (3) / have not been in use for years. (4) / No
error (5)

Ans (5): No error.


550. When I heard (1) / footsteps behind me (2) / I was being scared (3) / that I would be attacked. (4)

/No error (5)


Ans (3): It is incorrect to use Past Progressive (Passive) here. Hence, I was scared – should be used.
551. The Manager said that (1) / he wanted to known (2) / the pros and cons (3) / of the issue. (4) / No error (5)

Ans (2): Here, he wanted to know should be used.


552. Jason sell his house and (1) /moved to a flat (2) /because it was (3) /expensive to maintain. (4)/ No error (5).

Ans (1): Here, Past Simple i.e. John sold his house … should be used because the sentence shows Past time.
553. He accepted the offer (1) /because there was (2) /nothing more better (3) /coming his way. (4)/ No error (5).

Ans (3): Here, the error is of double comparatives. Hence, nothing better/nothing much better … should be used.
554. Most villages were cut off (1) /from the rest of (2) /the country (3) /owing to heavy floods. (4)/ No error (5).

Ans (1): Here, Most of the villages … should be used.


555. The trainer revised (1) /his work thorough (2) /so that no one (3) /could find fault with it.. (4)/ No error (5).

Ans (2): Here, Adverb i.e. his work thoroughly … should be used because an Adverb modifies a Verb.
556. The band will play (1) /the National Anthem when (2) /the President arrives (3) /on the country. (4)/ No error
(5).

Ans (4): Her, in the country … should be used.


557. If you are (1) /caught stealing (2) /in the super market (3) /you will be fined. (4) / No error (5)

Ans (5): No error


558. They left very early (1) /for the station (2) /for fear that thy might (3) / be held up on traffic. (4)/ No error (5).

Ans (4): Here, be struck in traffic or be held in traffic … should be used.


559. By the time the (1) /fire engine arrived (2) /at the scene. (3) /the building had already collapsing. (4)/ No error
(5).

Ans (4): The structure of a sentence in Past Perfect: Subject + had + V3. Here, the building had already collapsed …
should be used.
560. Gloria and Fiona (1) /are always in (2)/a state of disagreement (3) /for one another. (4)/ No error (5).

Ans (4): Here, with each other/one another … should be used.


561. The man which stole the camera (1) /hid it in his home (2) /until he could safely (3) /get it out of town. (4)/
No error (5).

Ans (1): Here, The man who stole … should be used because who is used for living beings.
562. There is a new shampoo (1)/in the market (2)/that is good for people (3)/with dry hairs. (4)/No error(5)

Ans (4): When ‘hair’ is used in general sense, it has no plural. When it is used in specific sense as countable Noun,
its plural is – hairs.
For example, His hair is dark.
There are only two grey hairs on his head now. Hence, with dry hair should be used here.
563. My teacher (1)/don’t hear me (2)/because she was (3)/playing the guitar. (4)/No error (5)

Ans (2): Here, Past Simple i.e. didn’t hear me should because the sentence showa past time.
564. We need (1)/another information (2)/ before we (3)/ reach a decision. (4)/No error

Ans (2): Here, more information should be used.


565. The visitor can choose (1)/either the normal train (2)/ and the express (3)/which stops only at major stations.
(4)/No error (5)

Ans (3): Here, or the express should be used. Either … or is correct form of correlative.
566. The government wish (1)/to raise (2)/the standard of football (3)/in the country. (4)/No error (5)

Ans (1): Here, subject is singular. Hence, The government wishes should be used.
567. In the recent years (1)/more and more violence (2)/were seen (3)/on television. (4)/No error (5)

Ans (3): The word ‘violence’ is an uncountable Noun. Hence, was seen should be used.
568. It gives me (1)/ great pleasure (2)/to sent you (3)/this little gift. (4)/No error (5)

Ans (3): Here, to send you should be used. The structure of an Infinitive is: to + V1 (Original Verb).
569. My grandfather (1)/does not know about (2)/my parents (3)/getting divorced. (4)/No error (5)

Ans (5): No error.


570. There are so many good shops (1)/that it is easiest (2)/for people to buy (3)/what they want. (4)/No error (5)

Ans (2): Here, that it is a easy / that it becomes easy should be used.
571. He wants to (1)/study further (2)/but does not (3)/want to leave his hometowns. (4).No error (5)

Ans (4): Here, want to leave his hometown should be used.


572. Farmers will be (1) / entitled to fresh (2) / agricultural loans (3) / from July onward. (4) / No error (5)

Ans (4): Replace group of words ‘from July onward’ by ‘from July onwards’. The word ‘onward’ is an Adjective
whereas the word ‘onwards’ is an Adverb. The meaning of ‘from … onwards’ is continuing from a particular time.
Look at the sentences:
They lived there from the 1990s onwards.
573. Today there is (1) / over one billion (2) / people living in poverty (3) / in developing countries.

(4) / No error (5)


Ans (1): Here, the subject ‘there’ will take Verb according to the number of object. Hence, today there are over one
billion people …’ will be correct sentence.
574. There is no need (1) / to import maize (2) / since there is (3) / any demand for it. (4)/ No error. (5)

Ans (4): Replace ‘any demand for it’ by ‘no demand for it’. The sense of the sentence tells us what to use.
575. Accordingly their policies (1) / banks write off (2) / outstanding amounts (3) / in NPA accounts.

(4) / No error (5)


Ans (1): The word ‘accordingly’ is an Adverb while the word ‘according to’ is a preposition. Hence, replace
‘accordingly their policies’ by ‘accordingly to their policies’.
Look at the sentences:
We have to discover his plans and act accordingly. He has been absent six times according to our records.
576. In order to (1) / be profitable (2) / a company needs (3) / to control its costs. (4) / No error. (5)

Ans (5): No error.


577. The idea behind the (1) / scheme is to encourage (2)/the private sector to (3) / invest in infrastructure. (4) No
error (5)

Ans (5): No error.


578. The company is (1) / expected to grow by (2) / as more as 9 percent (3) / during the next year. (4)

/ No error (5)
Ans (3): Replace ‘as more as 9 percent’ by ‘approximately 9 percent/about 9 percent/more that 9 percent’.
579. Due to varieties (1) / legal problems the (2) / plant is not (3) / fully operational. (4) / No error (5)

Ans (1): Replace ‘due to a variety’ by ‘due to a variety of’.


Look at the sentences:
There is a wide variety of patterns to choose from. He designed for a variety of reasons.
580. The meeting with (1) / employees will hold (2)/ once the management (3) / finalises the marger. (4)/ No error.
(5)

Ans (2): Here, use of Passive Voice is proper. Hence, the meeting with employees will be held
…will be a correct sentence.
581. The number of (1)/ATMs are (2) / being increased to (3) / serve customers better. (4) / No error (5)

Ans (1): Article ‘the’ has been used before ‘number’. Hence, singular Verb will be used.
582. Rajiv has won a prize of (1) /two million dollars which (2) /has to be shared (3) /with all his team members.
(4) / No error (5).

Ans (4): Groups of words ‘with all his team members’. Should be replaced by ‘among all his team members’.
Preposition ‘with’ has wrongly been used here.
583. I do not understand (1) /about how the payment (2) /was made without (3) /the manager’s permission. (4) /
No error (5).

Ans (2): Here, Preposition ‘about’ is Superfluous.


584. The company has (1) /decided to sell half (2) /of its shares (3) /to a Chinese firm. (4) / No error (5).

Ans (3): The possessive case of ‘company’ should be ‘its’. Hence of ‘its shares’ should be used.
585. By opening so many (1) /branches in such a (2) /short time, they have (3) /used the wrong strategy. (4) / No
error (5).

Ans (4): There is no definite thing. Hence, ‘used a wrong strategy’ should be used.
586. During the interview (1) /with the General Manager (2)/ I asked her what challenges (3) / she was faced. (4) /
No error (5).

Ans (4): Active Voice should be used. Hence, she faced will be the correct usage.
587. We have an account (1) /with this bank and (2) /find the staff much (3) / helpful and knowledgeable. (4) /No
error (5).

Ans (3): The structure of the sentence in Present Perfect Tense is:
Subject + have + has + V3
Hence, replace ‘find the staff much’ by ‘found the staff much’.
588. The scheme has been (1) /implemented only in these (2) /part of the country and the (3) /Prime Minister will
visit it shortly. (4) / No error (5).

Ans (2): Replace ‘implemented only these’ by ‘implemented only in this’. Here, singular sense is evident.
589. Today we are closer (1) /to reaching an agreement (2) /than we were (3)/ a few months ago. (4) / No error (5).

Ans (2): The form of infinitive is: to +V1. Hence, ‘to reach an agreement’ should be used.
590. We have been (1) /telling them to (2) /drive carefully but (3) /they are never listening. (4) /No error (5).

Ans (5): No error.


591. There is all sorts (1) /of regulations that have (2)/ to be met before (3) / we get a license. (4) /No error (5).

Ans (1): The word; Regulation’ is plural. Hence, ‘There are all sorts’ should be used.
592. The medias, (1)/ especially television, (2)/ has a very powerful (3)/ influence on children. (4)/ No Error (5)

Ans (1): The word ‘media’ is used in Singular/Plural sense. Its Plural is not ‘medias’
Look at the sentence:
The media was/were accused of influencing the final decision.
593. The courses / (1) in the field of psychology/ (2) should provide more/(3) practical sessions. (4)/ No Error (5)

Ans (1): Here, The course should be used. The word course means % a series of lessons or lectures on a Particular
subject.
594. Comparing to the other (1) / states in India (2)/ Goa is the (3) / smallest geographically. (4)/ No Error (5)

Ans (1): Here, Past Participle i.e. compared to the … should be used.
595. My friend (1)/ gift me two (2)/ cookery books, (3)/ which I find very useful. (4)/ No error (5)

Ans (2): Here, Present Perfect i.e. has gifted …. Should be used.
596. The dentist stressed (1) /on the importance (2)/ of brushing one’s teeth (3) /two time regularly. (4)/No error
(5)

Ans (4): Here, twice regularly … should be used


597. What are (1) the main ingredients (2) /in cooking (3) /South Indian dishes? (4)/ No Error (5)

Ans (3): Here, fro cooking … should be used.


Look at the sentence:
Coconut is a basic ingredient for many curries.
598. Last year there were (1)/ so many mangoes (2)/ that we had to give (3)/ the half .(4)/ No Error (5)

Ans (3): Here, and we had to give … should be used.


599. If you need someone (1)/ to helping you (2)/ I shall be free (3)/ all day tomorrow. (4)/ No Error (5)

Ans (2): Here, to help you should be used. To + V1 = infinitive.


600. Our mathematics teacher (1)/ does not allow us (2)/ to use a calculator (3)/ during her class. (4)

/No Error (5)


Ans (5): No error.
601. The water (1)/ in the cancel (2)/ was much more higher (3)/ than usual. (4)/ No Error (5)

Ans (3): Here, there is an error of double comparatives. Hence, was much deeper/higher … should be used here.
602. He is neither in (1)/favour of arms race (2)/or in favour of (3)/simple nuclear disarmament. (4)/ No error (5)

Ans (3): “Neither … nor” is correct connective.


603. Naturalisation is the (1)/process by which (2)/a immigrant becomes a citizen (3)/of his new country. (4)/No
error (5).

Ans (3): “An immigrant becomes …” is correct usages, as immigrant takes ‘an’ as determiner.
604. A high fat diet (1)/not only increases the risks (2)/of heart ailments (3)/however also that of other disorders.
(4)/No error (5)

Ans (4): “Not only … but also” is the correct form of connective.
605. When two vowel (1)/sounds occurs in direct succession, (2)/the transition between them (3)/is often difficult
to make. (4)/No error (5)

Ans (2): Here, the subject (when two vowel sounds) is Plural. Hence, it will take a Plural Verb.
For example,
Ram and Mohan play together.
↓↓
Plural Subject Plural Verb
606. One solutions is to (1)/avoid the impasse altogether, (2)/by taking a (3)/slightly different route. (4)/No error
(5)

Ans (1): ‘One solution is’ will be the correct usage, because one is singular.
607. She was running (1)/ a very high fever (2)/ and thus her mother takes (3)/ her to the doctor. (4)/ No error (5)

Ans (3): The principal Clause is in Past Tense. Hence, ‘and thus her mother takes’ (Simple Present) should be
replaced by ‘and so her mother took’ i.e. in Simple Past.
608. He talked on the phone (1)/ for hours together (2)/ who really irritated (3)/ his parents a lot. (4)/ No error (5).

Ans (3): Who is used to show which person or people you mean. Which is used to be exact about the thing or things
that you mean.
Look at the sentences: Houses which overlook the lake cost more. The people who called yesterday want to buy the
house. Therefore, ‘who really irritated’ should be replaced by ‘which really irritated’.
609. Although his speech (1)/ was not very clearly (2)/ everyone understood (3)/ the underlying meaning. (4)/ No
error (5)

Ans (2): Use of Adverb is incorrect as no verb is being modified. Hence, ‘was not very clear (Adjective)’ should be
used.
610. Despite working (1)/ very hard he (2)/ failed to achieve (3)/ the desiring results. (4)/ No error (5)

Ans (4): Replace ‘the desire results’ by ‘the desired (Adjective) results’.
611. He was very excited (1)/ about go to (2)/the park with (3)/ his younger brother. (4)/ No error (5)

Ans (2): Present Participle form should be used. Hence, replace ‘about go to’ by ‘about going to’.
612. Saddened by (1)/ the sudden demise of (2)/ his favourite pet he decided (3)/ no to go to the party. (4)/ No
error (5)

Ans (5): No error.


613. There is a ray of hope (1)/among the masses (2)/this year as more candidates (3)/ are being educated well.
(4)/ No error (5)

Ans (5): No error.


614. As she is late (1)/ for work she decided (2)/ to skip breakfast and (3)/ head directly to office. (4)/ No error (5)

Ans (1): As she was late … will be correct usage. The sense of the sentence suggests so.
615. In my visiting (1)/ to the country (2)/ I decided to go (3)/ on a tour all by myself. (4)/No error (5) Ans (1):
Replace ‘In my visiting’ by ‘During my visit’. During is used to say when something happens.

616. The tiger was not (1)/the only dangerous animal (2)/ in the forest (3)/ there was hyenas too. (4)/ No error (5)

Ans (4): Here Plural Verb should be used as hyenas is in Plural form.
617. There were (1)/many people (2)/present on the (3)/ award function. (4)/ No Error (5)

Ans (3): Replace present on with present at.


618. It was a long (1)/and uncomfortable journey (2)/ but he managed (3)/to reach with time. (4)/ No Error (5)

Ans (4): Replace to reach with time with to reach in time.


619. The car broke (1)/down while he (2)/was on his (3)/way to work. (4)/ No Error (5)

Ans (5): No error.


620. Do you wanted (1)/to discuss this (2)/project today or can (3)/we do it tomorrow? (4)/ No Error (5)

Ans (1): Do you wanted with Do you want. The structure of the sentence is: Do + S + V1 + O
For detailed explanation see Kiran’s Standard English Grammar.
621. Nina had applied (1)/for a loans for (2)/post-graduate studies but (3)/it was not sanctioned. (4)/ No Error (5)

Ans (2): Replace for a loans with for a loan.


622. People find it (1)/difficult to understand (2)/him as he (3)/don’t speak clearly. (4)/ No Error (5)

Ans (3): Replace him as he with as he.


623. Dr.Shah was explain (1)/the benefits of (2)/exercising atleast (3)/five times a week. (4)/ No Error (5)

Ans (1): Replace was explain with explains.


624. The actor donated (1)/ a huge sum of (2)/ the money to the (3)/ orphanage on Children’s Day. (4)/ No Error
(5)

Ans (3): Replace the money with money.


625. The customers were asked to (1)/ give their feedback after (2)/ using the try pack of the (3)/ new detergent
powder. (4)/ No Error (5)

Ans (3): Replace using the try pack with using the trial pack.
626. The road from the (1)/river bank to the hut (2)/ was more difficult than (3)/we have feared. (4)/No error. (5)

Ans (4): Use ‘We had feared’ in the place of ‘we have feared’. The sentence implies past time. Therefore, we should
use Past Perfect.
627. I found that (1)/ I had had the (2)/hardest time in writing dialogues (3)/for elderly people. (4)/No error (5)

Ans (5): No error.


628. During the course of the exhibition, (1)/the old man who was (2)/ looking usually cheerful (3)/ kept on take
notes. (4)/ No error (5)

Ans (4): In the given sentence Gerund will be used in the place of infinitive. The correct form will be Kept on taking
notes.
629. Unable to decide (1)/ what to do, (2)/ he stared at the fountain (3)/ at the centre of the park. (4)/ No error (5)

Ans (1): Being unable to decide … .is correct.


630. The bank employees could remember John (1)/as he has been maintained (2)/ his account with the bank (3)/
for the last ten years. (4)/ No error (5)

Ans (2): Here Past Perfect Continuous should be used … as had been maintaining … is correct.
631. There has not a (a)/ moment when the gates (2)/ of the bungalow (3)/ were not being watched (4)/ No error
(5)

Ans (1): There was not a moment … is a correct usage.


632. Haven’t you notice (1)/ that for the past one month (2)/ he hasn’t been here (3)/ at his regular place? (4)/ No
error (5)

Ans (1): The third form of Verb (V3) is in Interrogative of Past Perfect. Therefore, Haven’t you noticed …. Is correct
usage.
633. It seemed to him (1)/ that the manager of the (2)/ bank looking at him (3)/ in a rather suspicious manner. (4)/
No error (5)

Ans (3): Auxiliary Verb ‘was’ should be used in the sentence. Therefore… The bank was looking at him …. Correct
usage.
634. The policeman went away, (1)/ apparently convincing (2)/ but he did not (3)/ feel safe at all. (4)/ No error (5)

Ans (2): The policeman went away apparently convinced … is correct. The use of Gerund
convincing is incorrect.
635. It was with that moment (1)/ that the front door scanner (2)/ warned us that (3)/ someone was approaching.
(4)/ No error (5)

Ans (1): The use of Preposition ‘with’ is incorrect. Therefore, it was at that moment …. Is correct.
636. We all seemed to forget (1)/ for a while that (2)/ something very strangely and terribly (3)/ had happened. (4)
/ No error (5)

Ans (3: he use of Adverb is incorrect. Therefore… Something very strange and terrible (Adjectives) …. Is correct
sentence.
637. He is one (1)/ of the religious reformers (2)/ who has spoken out (3)/ strongly against casteism (4)/ No error
(5)

Ans (3): Consider Noun/Pronoun used after ‘One of” as Antecedent and use Verb accordingly.
For example,
Dryden remains one of those who

Plural have set standards fro English verse.

Plural
Therefore, He is one of the religious reforms who have spoken out …. Is correct sentence.
638. I becoming an administrator (1)/ was not liked (2) by my parents (3)/ and our well wishers. (4)/ No error (5)

Ans (1): My becoming an administrator is correct.


639. With his many talents (1)/ and dynamic leadership (2)/ he served his country (3)/ in commendable and lasting
ways. (4)/ No error (5)

Ans (5): No error


640. He was not (1)/ a leader who (2)/ was desiring of making (3)/ compromises with the rulers. (4)/No error (5)

Ans (3): He was not a leader who desired for …. Is correct. Here, Simple Past should be used.
641. Woman after women (1)/claimed to speak against (2)/ the present political situation (3) / of Bihar in the
meeting. (4) / No error (5)

Ans (1): ‘Women after women’ should be replaced with ‘women after woman’ because if the structure of the
sentence is: Noun + Preposition + Similar Noun, then both the Nouns should be in Singular Number
For example,
Women after woman delivered speech. (√) Women after women delivered speech. (X)
642. More than one student (1)/ have passed the examination (2)/ of the first terminal test (3)/ of pronoun. (4)/ No
error (5)

Ans (2): Use ‘has’ in the place of ‘have’.


Consider the following structure.
More than one + Singular Noun + Singular verb. (√)
More + PCN = than one + PV (√) For example,
More than one classroom is vacant. (√) More than one classroom are vacant. (X)
643. If Sunit would have (1)/solving this, he would have (2) /got a reward (3) / in the debate. (4) / No error (5)

Ans (1): We should use ‘had’ in the place of ‘would have’ because the structure of Past Conditional sentence would
be as follows:
If + S + had + V3 + would have +V3 or
Had + S + V3 + S would have + V3
If Rashmi had asked me, I would have helped her. (√) Or, Had Rashmi asked me, I would have helped her. (√)
If Rashmi would have asked me , I would have helped her.
644. One should (1)/ be honest (2) / to his word (3) / in all situations. (4) / No error (5)

Ans (3): Use ‘to one’s word’ in the place of ‘to his word’. When ‘One’ is used in Nominative Case, then its form will
be ‘one’ and in Possessive Case, it should be ‘one’s’.
For example,
One should do one’s duty.
645. When the house was (1)/ set on the fire (2)/ all the neighboring people (3) / ran and started extinguishing it.
(4)/ No error (5)

Ans (2): Use ‘set on fire’ in the place of ‘set on the fire’.
‘Set on the fire’ is an idiomatic expression; therefore, Article will not be used. For example,
Huts were set on the fire. (X) Huts were set bon fire. (√)
646. By this time next year (1)/ Mritunjay has had (2)/ settled in Buxar (3) / with his parents. (4)/ No error (5)

Ans (2): The given sentence is based on the structure of Future Perfect Tense. Therefore, use ‘Mritunjay will have’ in
the place of ‘Mritunjay has had’.
For example,
By this time next month he will have returned. (√)
By this time tomorrow Babita and jyoti will have reached Kathiar. (√)
647. All know (1)/ that Rani is (2)/ the most unique speaker (3) / of her class. (4)/ No error (5)

Ans (3): Use ‘a unique speaker’ in the place of ‘next most unique speaker’. There are some Adjectives which convey
the meaning of the Superlative Degree even if these are used as Positive Degree.
For example,
It is the most excellent toy. (X) It is an excellent toy, (√)
Sainky is the most unique boy. (X) Sainky is a unique boy. (√)
648. Being a rainy day (1)/ Ashis decided to stay indoors (2)/ and enjoy himself (3)/ with music. (4)/ No error (5)

Ans (3): Use ‘it being a rainy day’ in the place of ‘Being a rainy day’. When Participle is used, it takes a subject
which is called subject of reference.
For example,
Being a fine morning, the children went to the field. (X) It being a fine morning, the children went to the field. (√)
649. I advised Prashant (1)/ not to walk (2)/ on the road (3)/ lately at night. (4)/ No error (5)

Ans (4): Use ‘late at night’ in the place of ‘Lately at night’.


Lately means recently and Late means after fixed time. For example,
Sarika come late Sarika came lately.

Вам также может понравиться